2014 musculoskeletal tumors and diseases

102
MUSCULOSKELETAL TUMORS AND DISEASES Self-Scored Self-Assessment Examination 2014

Upload: nanda-gema

Post on 13-Dec-2015

19 views

Category:

Documents


1 download

DESCRIPTION

MUSCULOSKELETAL TUMORS AND DISEASESSelf-Scored Self-Assessment Examination2014

TRANSCRIPT

Page 1: 2014 Musculoskeletal Tumors and Diseases

MUSCULOSKELETAL TUMORS AND DISEASES

Self-Scored Self-Assessment Examination

2014

Page 2: 2014 Musculoskeletal Tumors and Diseases

Musculoskeletal Tumors and Diseases Self-Scored Self-Assessment Examination 2014

Figure 1a

Figure 1b

Figure 1c

CLINICAL SITUATION FOR QUESTIONS 1 THROUGH 4

Figures 1a through 1c are the radiograph, CT scan, and bone scan of a 49-year-old woman

admitted to the hospital with right hip pain and an inability to bear weight for 2 days. She

states that the pain began 3 months ago, but suddenly progressed just before her admission.

Question 1 of 143

You decide to obtain further imaging. The best test to order would be

1- ultrasound of the thyroid.

2- MRI scan of the pelvis.

3- CT scan of the chest, abdomen, and pelvis.

4- indium-labeled bone scan.

PREFERRED RESPONSE: 3- CT scan of the chest, abdomen, and pelvis.

Question 2 of 143

Given the scenario, what is the most likely finding on histopathologic review?

1- Metastatic adenocarcinoma

2- Ewing sarcoma

3- Lymphoma of bone

4- Hip joint sepsis

PREFERRED RESPONSE: 1- Metastatic adenocarcinoma

Question 3 of 143

The preferred surgical intervention for this fracture is

1- Radical resection and reconstruction

2- Intramedullary nail fixation

Page 3: 2014 Musculoskeletal Tumors and Diseases

3- Cannulated screw fixation

4- Complex total hip arthroplasty

PREFERRED RESPONSE: 4- Complex total hip arthroplasty

Question 4 of 143

External beam radiotherapy

1- should include the instrumented femur and periacetabular area.

2- should include the femur only.

3- should include the acetabulum only.

4- is contraindicated for this patient.

PREFERRED RESPONSE: 1- should include the instrumented femur and periacetabular

area.

DISCUSSION

The occurrence of a femoral neck fracture in a relatively young patient with no history of

trauma should alert the physician to potential for an underlying pathological process. The

most likely culprit would be metastatic disease, myeloma, or lymphoma. Systemic imaging is

undertaken to determine whether a primary site or site(s) of metastasis can be discovered.

Only after obtaining tissue can the definitive diagnosis be made. Common primary

malignancies that metastasize to bone are breast, lung, prostate, renal, and thyroid

carcinomas. Multiple myeloma and lymphomas may also have a similar appearance in the

bone. In this instance, not only is the femoral neck fractured, but there is a substantial lytic

process in the acetabulum as well as other skeletal sites of involvement. Therefore, complex

hip arthroplasty is the best choice for surgical intervention. Radiotherapy is indicated in the

care of this patient as well. Generally, the radiation field should include the entirety of the

involved implants plus a margin of tissue around them. In this case, this would include the

entire progression of instrumented femur and the periacetabular area. This approach

minimizes risk for local recurrence and transplanted tumor during instrumentation and/or as a

result of postoperative ecchymosis or bleeding. There is no indication for preoperative

radiation in this scenario.

RECOMMENDED READINGS

Buecker PJ, Rothrock CP. Current management of skeletal metastasis. Orthopaedic Knowledge Online Journal. 2010.

Rougraff BT, Kneisl JS, Simon MA. Skeletal metastases of unknown origin. A prospective study of a diagnostic strategy. J Bone Joint Surg

Am. 1993 Sep;75(9):1276-81. PubMed PMID: 8408149. View Abstract at PubMed

Question 5 of 143

Li-Fraumeni syndrome (LFS) is associated with

Page 4: 2014 Musculoskeletal Tumors and Diseases

1- multiple hemangiomas.

2- multiple hereditary osteochondromatosis.

3- soft-tissue sarcomas.

4- neurofibromatosis.

PREFERRED RESPONSE: 3- soft-tissue sarcomas.

DISCUSSION

LFS is a rare genetic condition characterized by an increased risk for developing multiple

types of cancer. The most common types of cancers associated with LFS include soft-tissue

sarcomas (tumors in fat, muscle, nerves, joints, blood vessels, bones, or deep skin), breast

cancer, leukemia, lung cancer, and brain tumors. The cancers that occur in LFS can be

diagnosed during childhood, adolescence, or adulthood. Most people with LFS are found to

have mutations in the TP53 gene. Multiple hemangiomas, osteochondromas, and

neurofibromatosis are not associated with LFS.

RECOMMENDED READINGS

Chompret A. The Li-Fraumeni syndrome. Biochimie. 2002 Jan;84(1):75-82. PubMed PMID: 11900879.

View Abstract at PubMed

Moule RN, Jhavar SG, Eeles RA. Genotype phenotype correlation in Li-Fraumeni syndrome kindreds and its implications for management.

Fam Cancer. 2006;5(2):129-33. Review. PubMed PMID: 16736281.

View Abstract at PubMed

Olivier M, Goldgar DE, Sodha N, Ohgaki H, Kleihues P, Hainaut P, Eeles RA. Li-Fraumeni and related syndromes: correlation between tumor type, family structure, and TP53 genotype. Cancer Res. 2003 Oct 15;63(20):6643-50. PubMed PMID: 14583457.

View Abstract at PubMed

Wong P, Verselis SJ, Garber JE, Schneider K, DiGianni L, Stockwell DH, Li FP, Syngal S. Prevalence of early onset colorectal cancer in

397 patients with classic Li-Fraumeni syndrome. Gastroenterology. 2006 Jan;130(1):73-9. PubMed PMID: 16401470. View Abstract at PubMed

Question 6 of 143

A 60-year-old woman has a proximal femur fracture. A permeative, lytic defect is recognized

at the fracture site. Appropriate imaging studies are performed and show no other lesions.

What is the next treatment step?

1- Cephalomedullary nail

2- Standard antegrade intramedullary nail

3- Resection and arthroplasty reconstruction

4- Open biopsy

PREFERRED RESPONSE: 4- Open biopsy

DISCUSSION

In this patient, tissue diagnosis should be obtained prior to any surgical intervention to avoid

unnecessary contamination in the event the lesion is a sarcoma. Open biopsy will yield the

diagnosis in the majority of cases. The specific choice for surgical treatment of a proximal

Page 5: 2014 Musculoskeletal Tumors and Diseases

femur pathologic fracture is controversial, but it would typically involve either a

cephalomedullary nail or resection/reconstruction. A simple antegrade femoral nail rarely is

sufficient in this setting.

RECOMMENDED READINGS

Rougraff BT, Kneisl JS, Simon MA. Skeletal metastases of unknown origin. A prospective study of a diagnostic strategy. J Bone Joint Surg

Am. 1993 Sep;75(9):1276-81. PubMed PMID: 8408149. View Abstract at PubMed

Pap DP, Mayerson JL, Scharschmidt TJ. Soft tissue tumor recurrence after intramedullary stabilization of impending pathologic fractures. Orthopedics. 2012 Mar 7;35(3):e448-52. doi: 10.3928/01477447-20120222-43. PubMed PMID: 22385463.

View Abstract at PubMed

Figure 7

CLINICAL SITUATION FOR QUESTIONS 7 THROUGH 9

Figure 7 is the MRI scan of a 58-year-old woman who is being evaluated for knee

replacement. Examination reveals a large painless mobile lump on her anterior thigh. The

patient reports the lump has been there for at least 3 years, and she does not know if it is

growing or if it should be removed.

Question 7 of 143

What feature of the lesion most eliminates the need for biopsy prior to resection?

1- Painless nature

2- Location

3- Appearance on MRI scan

4- Rate of growth

PREFERRED RESPONSE: 3- Appearance on MRI scan

Question 8 of 143

Atypical lipomatous lesions often appear as benign lipomas. What study will distinguish an

atypical lipomatous lesion?

1- Cytogenetic testing showing evidence of a ring chromosome (MDM 2)

2- MRI scan showing an entire lesion made up of material isointense to fat on all sequences

Page 6: 2014 Musculoskeletal Tumors and Diseases

3- Hematoxylin and eosin staining showing mature adipocytes

4- Immunohistochemistry staining for BCL-2

PREFERRED RESPONSE: 1- Cytogenetic testing showing evidence of a ring chromosome

(MDM 2)

Question 9 of 143

The pathology is consistent with atypical lipomatous tumor (ALT) well-differentiated

lipoma-like liposarcoma (WDLLS). What is the appropriate treatment and prognosis for this

condition?

1- Wide excision with a high chance of local recurrence and moderate chance of metastasis

2- Marginal excision with a moderate chance of local recurrence and low chance of

metastasis

3- Intralesional excision with a low chance of local recurrence and low chance of metastasis

4- Intralesional excision with a low chance of local recurrence and high chance of metastasis

PREFERRED RESPONSE: 2- Marginal excision with a moderate chance of local recurrence

and low chance of metastasis

DISCUSSION

ALTs are the extremity equivalent to a well-differentiated liposarcoma (WDLS) of the

retroperitoneum and other visceral locations. ALTs can pose a difficult diagnostic dilemma.

They often appear as lipomas, characterized as being made up entirely of fat, isointense to fat

on all MR scan sequences without heterogeneity. Atypical features may be increased

thickened internal septation, large and deep nature, and enhancement on post-gadolinium

studies. With no heterogeneous portions on MRI scan, biopsy is not needed because it is

unlikely to differentiate lipoma from ALT. The painless nature or superficial locations are not

reliable indicators of benign lesions. Concurrent surgery is never an appropriate reason for a

lesion to be removed unless previous imaging shows otherwise.

Benign lipomas and ALT may have some similarities. They both appear on MRI scan to be

made of fat with identical signal characteristics. Routine histology will show mature

adipocytes in both with minimal atypia in the ALTs. Cytogenetic testing, however, will show

a supernumerary ring and/or giant rod chromosomes in ALT, with amplification of the

MDM2 oncogene. BCL-2 is the B-cell lymphoma apoptosis regulator protein, seen in

lymphoma as well as some leukemias and carcinomas.

ALT or WDLLS are lesions with a moderate chance of local recurrence—certainly higher

than for ordinary lipoma, with reported rates of up to 50%. They pose low, if any, distant

metastatic potential. For this reason, marginal excision is recommended. When these lesions

are locally recurrent, they most likely are of the same histology; however, a small percentage

can return as a higher-grade malignancy (with an associated increase in metastatic potential).

The other concern regarding ALTs is the reported up to 10% incidence of dedifferentiation

within them, underscoring the recommendation for excision.

Page 7: 2014 Musculoskeletal Tumors and Diseases

RECOMMENDED READINGS

Conyers R, Young S, Thomas DM. Liposarcoma: molecular genetics and therapeutics. Sarcoma. 2011;2011:483154. doi:

10.1155/2011/483154. Epub 2010 Dec 27. PubMed PMID: 21253554.

View Abstract at PubMed

Hopyans S, Wunder JS, Randall RL. Molecular biology in musculoskeletal neoplasia. In: Schwartz H, ed. Orthopaedic Knowledge Update:

Musculoskeletal Tumors 2. Rosemont, IL: American Academy of Orthopaedic Surgeons; 2007:383-392.

Gaskin CM, Helms CA. Lipomas, lipoma variants, and well-differentiated liposarcomas (atypical lipomas): results of MRI evaluations of 126 consecutive fatty masses. AJR Am J Roentgenol. 2004 Mar;182(3):733-9. PubMed PMID: 14975977.

View Abstract at PubMed

Pilotti S, Della Torre G, Mezzelani A, Tamborini E, Azzarelli A, Sozzi G, Pierotti MA. The expression of MDM2/CDK4 gene product in

the differential diagnosis of well differentiated liposarcoma and large deep-seated lipoma. Br J Cancer. 2000 Apr;82(7):1271-5. PubMed

PMID: 10755400. View Abstract at PubMed

RESPONSES FOR QUESTIONS 10 THROUGH 12

1- Wide resection alone

2- Amputation

3- Chemotherapy and external beam radiation

4- External beam radiation and surgical resection

Match the preferred treatment option above with the patient description below.

Question 10 of 143

A 70-year-old woman has a 15-cm subfascial soft-tissue sarcoma in her left thigh with no

signs of metastatic disease on staging studies.

1- Wide resection alone

2- Amputation

3- Chemotherapy and external beam radiation

4- External beam radiation and surgical resection

PREFERRED RESPONSE: 4- External beam radiation and surgical resection

Question 11 of 143

A 57-year-old woman has a 3-cm low-grade subcutaneous sarcoma of the upper arm.

1- Wide resection alone

2- Amputation

3- Chemotherapy and external beam radiation

4- External beam radiation and surgical resection

PREFERRED RESPONSE: 1- Wide resection alone

Page 8: 2014 Musculoskeletal Tumors and Diseases

Question 12 of 143

A 33-year-old man has an unplanned resection of a high-grade pleomorphic sarcoma of the

hand with multiple positive microscopic margins on final pathology.

1- Wide resection alone

2- Amputation

3- Chemotherapy and external beam radiation

4- External beam radiation and surgical resection

PREFERRED RESPONSE: 2- Amputation

DISCUSSION

The management of soft-tissue sarcoma can present unique challenges. A systematic

approach is necessary if undue complications and problems are to be avoided. In general,

low-grade lesions smaller than 5 cm that are suprafascial can be excised with appropriate

margins without adjuvant treatment. Lesions that are 5 cm or larger, are high-grade, and deep

to fascia are most often treated with a combination of radiation and surgery. In the event a

sarcoma is removed with inadequate margins (a scenario that is usually preceded by

insufficient imaging before surgery), then the premium is on secondarily obtaining

appropriate margins, most commonly with re-resection. However, in the case of a lesion in

the hand or foot, multiple compartment contamination may require amputation to achieve

adequate local control.

RECOMMENDED READINGS

Cheng EY. Surgical management of sarcomas. Hematol Oncol Clin North Am. 2005 Jun;19(3):451-70, v. Review. PubMed PMID:

15939191. View Abstract at PubMed

Hohenberger P, Wysocki WM. Neoadjuvant treatment of locally advanced soft tissue sarcoma of the limbs: which treatment to choose? Oncologist. 2008 Feb;13(2):175-86. doi: 10.1634/theoncologist.2007-0165. Review. PubMed PMID: 18305063.

View Abstract at PubMed

Hueman MT, Thornton K, Herman JM, Ahuja N. Management of extremity soft tissue sarcomas. Surg Clin North Am. 2008 Jun;88(3):539-

57, vi. doi: 10.1016/j.suc.2008.04.003. Review. PubMed PMID: 18514697.

View Abstract at PubMed

Kaushal A, Citrin D. The role of radiation therapy in the management of sarcomas. Surg Clin North Am. 2008 Jun;88(3):629-46, viii. doi:

10.1016/j.suc.2008.03.005. Review. PubMed PMID: 18514703. View Abstract at PubMed

Figure 13a

Figure 13b

CLINICAL SITUATION FOR QUESTIONS 13 THROUGH 14

Page 9: 2014 Musculoskeletal Tumors and Diseases

Figures 13a and 13b are the MRI scans and biopsy specimen of a 64-year-old man who has

coccygeal pain.

Question 13 of 143

What is the most likely diagnosis?

1- Metastatic carcinoma

2- Giant-cell tumor

3- Chordoma

4- Chondrosarcoma

PREFERRED RESPONSE: 3- Chordoma

Question 14 of 143

Based on the images and histopathology, how is this patient best treated?

1- Chemotherapy and external beam radiotherapy

2- Resection

3- Resection and chemotherapy

4- External beam radiation alone

PREFERRED RESPONSE: 2- Resection

DISCUSSION

The imaging demonstrates a lobular T2 hyperintense sacral lesion, which on biopsy shows

physaliferous cells diagnostic of a sacrococcygeal chordoma. The histology differentiates this

from metastatic carcinoma, giant-cell tumor, or chondrosarcoma. Preferred treatment is en

bloc surgical resection. Although some centers employ adjuvant radiotherapy, this is of

unproven benefit at this time. Evolving data suggest benefits of particle beam therapy (proton

or carbon ion) from phase II studies, but this has not yet proven superior to en bloc surgery.

The lesion shown is surgically accessible.

RECOMMENDED READINGS

DeLaney TF, Liebsch NJ, Pedlow FX, Adams J, Dean S, Yeap BY, McManus P, Rosenberg AE, Nielsen GP, Harmon DC, Spiro IJ, Raskin

KA, Suit HD, Yoon SS, Hornicek FJ. Phase II study of high-dose photon/proton radiotherapy in the management of spine sarcomas. Int J Radiat Oncol Biol Phys. 2009 Jul 1;74(3):732-9. doi: 10.1016/j.ijrobp.2008.08.058. Epub 2008 Dec 25. PubMed PMID: 19095372.

View Abstract at PubMed

Fuchs B, Dickey ID, Yaszemski MJ, Inwards CY, Sim FH. Operative management of sacral chordoma. J Bone Joint Surg Am. 2005

Oct;87(10):2211-6. PubMed PMID: 16203885.

View Abstract at PubMed

Hulen CA, Temple HT, Fox WP, Sama AA, Green BA, Eismont FJ. Oncologic and functional outcome following sacrectomy for sacral

chordoma. J Bone Joint Surg Am. 2006 Jul;88(7):1532-9. PubMed PMID: 16818979. View Abstract at PubMed

Page 10: 2014 Musculoskeletal Tumors and Diseases

Figure 15a

Figure 15b

Figure 15c

Figure 15d

Figure 15e

Figure 15f

Figure 15g

Question 15 of 143

A 56-year-old podiatrist with a negative past medical history had anterior knee pain after an

injury. His radiographs, CT scan, and T1-weighted sagittal and fat-saturated axial MR images

are shown in Figures 15a through 15e, respectively. After arthroscopic partial medial

menisectomy, the patient was turned to the prone position and an open posterior arthrotomy

and excision was performed. Low-power and high-power hematoxylin and eosin stained

histologic specimens are shown in Figures 15f and 15g, respectively. Based on the history,

radiographs, CT scan, MRI scans, and histologic findings, what is the most likely diagnosis?

1- Localized pigmented villonodular synovitis (PVNS)

2- Biphasic synovial sarcoma

3- Nodular fasciitis

4- Synovial hemangioma

PREFERRED RESPONSE: 4- Synovial hemangioma

DISCUSSION

The incidental finding in the posterior intercondylar notch of the knee in this patient

represents a benign synovial hemangioma. No intralesional calcifications are shown either on

the plain radiographs or CT scan. The MRI scans reveal a hypervascular lesion with multiple

filling defects, with hyperintensity on T2-weighted images and low to intermediate signal

intensity on T1-weighted images. Histologically, vascular lakes within fine capillaries with a

synovium on the surface of the lesion are characteristic of this condition. Many patients with

synovial hemangioma present with pain, swelling, stiffness, or mechanical symptoms. As

with PVNS, the disease can be localized or diffuse in nature. Surgical excision, either open or

arthroscopic, is the recommended treatment. PVNS is the most common intra-articular tumor,

but hypointensity in either the diffuse or localized type is characteristic on MRI scan in both

T1- and T2-weighted images. Synovial sarcoma, although often found close to a joint, is not

characteristically found within a joint. Nodular fasciitis is not found in intra-articular

locations.

RECOMMENDED READINGS

Adelani MA, Wupperman RM, Holt GE. Benign synovial disorders. J Am Acad Orthop Surg. 2008 May;16(5):268-75. Review. PubMed

PMID: 18460687.

View Abstract at PubMed

Page 11: 2014 Musculoskeletal Tumors and Diseases

Weiss SW, Goldblum JR. Benign tumors and tumor-like lesions of blood vessels. In: Weiss SW, Goldblum JR, eds. Soft Tissue Tumors. 5th

ed. Philadelphia, PA: Mosby Elsevier; 2008:664-665.

Figure 16a

Figure 16b

Figure 16c

CLINICAL SITUATION FOR QUESTIONS 16 THROUGH 19

Figures 16a and 16b are the right femur radiographs of a 59-year-old man who has severe

pain in his right distal thigh and knee with no significant trauma and an inability to bear

weight. Blood tests demonstrate anemia, serum protein electrophoresis/urine protein

electrophoresis findings are negative, and electrolyte levels are within defined limits.

Question 16 of 143

Based on this patient’s history and imaging, the best next treatment step should include

1- immediate internal fixation of the fracture.

2- a skeletal survey.

3- a dual-energy radiograph absorptiometry (DEXA) scan.

4- a CT scan of the chest, abdomen, and pelvis with a whole-body bone scan.

PREFERRED RESPONSE: 4- a CT scan of the chest, abdomen, and pelvis with a whole-

body bone scan.

Question 17 of 143

No additional bony lesions were noted. What is the most appropriate next step?

1- Bone marrow biopsy

2- Immediate internal fixation of the fracture

3- Biopsy of the fracture site

4- Chemotherapy initiation

PREFERRED RESPONSE: 3- Biopsy of the fracture site

Page 12: 2014 Musculoskeletal Tumors and Diseases

Question 18 of 143

The final results of the patient’s presurgical studies reveal a large solid mass in the right

kidney. He was brought to the operating room and a frozen section of his femur revealed the

findings seen in Figure 16c. What is the best treatment option in this setting?

1- External fixation

2- Retrograde intramedullary rod

3- Distal femoral resection with megaprosthesis

4- Locking plate fixation with allograft of the defect

PREFERRED RESPONSE: 3- Distal femoral resection with megaprosthesis

Question 19 of 143

Approximately what percentage of the time does an unknown primary cancer get identified as

part of a full metastatic work-up that includes radiographs; blood tests; a CT scan of the

chest, abdomen and pelvis; whole-body bone scan; and biopsy of the metastatic focus?

1- 45%

2- 65%

3- 85%

4- 100%

PREFERRED RESPONSE: 3- 85%

DISCUSSION

Metastatic bone cancer is common, affecting nearly 5.3% of the nearly 5 million people with

cancer in the United States each year. This is a typical presentation of a patient with a newly

diagnosed pathologic femur fracture. Without a known history of cancer, this presentation

should initiate a complete metastatic work-up including a radiograph of the pathologic site;

blood tests; whole-body bone scan; and a CT scan of the chest, abdomen, and pelvis.

Additional imaging at the site of concern such as an MRI scan or CT scan may be helpful in

certain cases. The next step is to obtain a biopsy of lesional tissue before proceeding to

immediate fixation. However, it is important to note that this lesion may be a primary cancer

for which case resection or amputation is necessary. A skeletal survey would be helpful in the

setting of multiple myeloma. A DEXA scan has no value in this clinical setting. A bone

marrow biopsy should be considered in the case of myeloma, but not metastatic carcinoma.

Chemotherapy cannot be initiated in any case without a tissue diagnosis. The knowledge that

the patient was found to have a large solid renal mass is highly suspicious for renal cell

carcinoma, which is among the 5 most common primary carcinomas to metastasize to the

bone (breast, lung, thyroid, kidney, and prostate).

Treatment of the femur should focus on a robust repair (whether biologic or not) to outlive

the patient. A distal femoral resection with megaprosthesis is the most durable option for

periarticular renal cell carcinoma metastasis, which is known to respond poorly to

radiotherapy. The pathology shown is classic for a clear cell carcinoma of the kidney. In

Page 13: 2014 Musculoskeletal Tumors and Diseases

current practice, most primary cancers are identified as the result of a metastatic work-up.

Approximately 85% to 90% of primaries are determined from the history, physical, blood

tests, imaging, and biopsy results.

RECOMMENDED READINGS

Rougraff BT. Evaluation of the patient with carcinoma of unknown origin metastatic to bone. Clin Orthop Relat Res. 2003 Oct;(415

Suppl):S105-9. Review. PubMed PMID: 14600599. View Abstract at PubMed

Rougraff BT, Kneisl JS, Simon MA. Skeletal metastases of unknown origin. A prospective study of a diagnostic strategy. J Bone Joint Surg Am. 1993 Sep;75(9):1276-81. PubMed PMID: 8408149.

View Abstract at PubMed

Bickels J, Dadia S, Lidar Z. Surgical management of metastatic bone disease. J Bone Joint Surg Am. 2009 Jun;91(6):1503-16. doi:

10.2106/JBJS.H.00175. Review. PubMed PMID: 19487532.

View Abstract at PubMed

Biermann JS, Holt GE, Lewis VO, Schwartz HS, Yaszemski MJ. Metastatic bone disease: diagnosis, evaluation, and treatment. J Bone Joint

Surg Am. 2009 Jun;91(6):1518-30. Review. PubMed PMID: 19487533. View Abstract at PubMed

Lin PP, Mirza AN, Lewis VO, Cannon CP, Tu SM, Tannir NM, Yasko AW. Patient survival after surgery for osseous metastases from renal cell carcinoma. J Bone Joint Surg Am. 2007 Aug;89(8):1794-801. PubMed PMID: 17671020.

View Abstract at PubMed

Schulman KL, Kohles J. Economic burden of metastatic bone disease in the U.S. Cancer. 2007 Jun 1;109(11):2334-42. PubMed PMID:

17450591.

View Abstract at PubMed

Figure 20a

Figure 20b

Figure 20c

Figure 20d

Figure 20e

CLINICAL SITUATION FOR QUESTIONS 20 THROUGH 23

Figures 20a and 20b are the radiographs of an 83-year-old active, independent, and healthy

woman who has experienced 2 months of right lower thigh and knee pain. Her pain increased

progressively over the course of several weeks. While exiting a car she “bumped” her knee

against the door, felt a “crack,” and developed excruciating pain. She could no longer

ambulate and was brought to the hospital.

Question 20 of 143

Based on imaging alone, what does this bone lesion most closely resemble?

Page 14: 2014 Musculoskeletal Tumors and Diseases

1- Multiple myeloma

2- Nonossifying fibroma

3- Metastatic renal cell carcinoma

4- Enchondroma

PREFERRED RESPONSE: 4- Enchondroma

Question 21 of 143

Figures 20c through 20e show the biopsy performed of the above-noted lesion. Based on the

clinical history, radiograph, and biopsy, what diagnosis is most likely?

1- Enchondroma

2- High-grade osteosarcoma

3- Metastatic renal cell carcinoma

4- Dedifferentiated chondrosarcoma

PREFERRED RESPONSE: 4- Dedifferentiated chondrosarcoma

Question 22 of 143

What treatment option carries the lowest risk for local recurrence?

1- Excision of tumor followed by open reduction with internal fixation

2- Megaprosthetic reconstruction of the femur and knee after tumor resection

3- Above-the-knee amputation with wide surgical margin

4- Revision total knee arthroplasty with long-cemented stem after tumor resection

PREFERRED RESPONSE: 3- Above-the-knee amputation with wide surgical margin

Question 23 of 143

What is the most likely site of metastatic disease in patients with this diagnosis?

1- Liver

2- Lungs

3- Brain

4- Kidneys

PREFERRED RESPONSE: 2- Lungs

DISCUSSION

Page 15: 2014 Musculoskeletal Tumors and Diseases

This patient has a pathologic femur fracture. Her history suggests that her pain was

emanating from an impending fracture. The initial radiograph reveals a well-mineralized

lesion within the intramedullary canal with punctate calcifications. This by itself suggests an

enchondroma or low-grade chondroid lesion. It is important to note that enchondromas in the

long bones rarely cause pathologic fracture. That is not the case when they are present in the

hands and feet, where biologically they act more aggressively. Typically, a more aggressive

appearance such as cortical thinning or breach, soft-tissue mass, or periosteal elevation

suggests a chondrosarcoma. Unfortunately, a pathologic fracture makes the images somewhat

more difficult to interpret, which was the case with her MRI and CT scans. There is no

evidence from her radiographs that this is anything but a chondroid tumor. Multiple myeloma

often involves a radiolucent “punched out” lesion. Nonossifying fibromas are seen in children

and are well circumscribed, eccentric radiolucent lesions. Metastatic renal cell carcinoma

rarely demonstrates matrix on a radiograph. The histopathology clearly shows a cartilage-

based tumor. The chondroid tissue appears low grade. However, in an adjacent region there is

a high-grade component seen both on the low power view and the second higher-power

sample that does not resemble cartilage. This is highly suggestive of a dedifferentiated

chondrosarcoma. This signifies that a portion of the original tumor (low-grade

chondrosarcoma), which resembled the tissue of origin, converted into a different cell lineage

with more aggressive properties (nearly any type of high-grade sarcoma). These dangerous

tumors are highly malignant, with average 5-year survivals of about 50%. Local recurrence in

the setting of a pathologic fracture will be high secondary to spillage of tumor into the

fracture hematoma and disturbed compartments. In this case, an above-the-knee amputation

with a wide surgical margin would offer the lowest risk for local recurrence, but at the

expense of the most functional loss, especially in this age group. The most likely site of

metastatic disease from nearly any sarcoma is the lungs. Therefore, patients have their lungs

surveyed with at least a chest radiograph or, in most cases, a CT scan.

RECOMMENDED READINGS

Mavrogenis AF, Gambarotti M, Angelini A, Palmerini E, Staals EL, Ruggieri P, Papagelopoulos PJ. Chondrosarcomas revisited. Orthopedics. 2012 Mar 7;35(3):e379-90. doi: 10.3928/01477447-20120222-30. PubMed PMID: 22385450.

View Abstract at PubMed

Flemming DJ, Murphey MD. Enchondroma and chondrosarcoma. Semin Musculoskelet Radiol. 2000;4(1):59-71. Review. PubMed PMID:

11061692.

View Abstract at PubMed

Figure

24a

Figure

Figure

24c

Figure

24d

Figure

24e

Figure

24f

Figure

24g

Figure

24h

Figure

24i

Page 16: 2014 Musculoskeletal Tumors and Diseases

24b

Question 24 of 143

A 14-year-old boy has had wrist pain for 3 weeks. Radiographs are shown in Figures 24a and

24b. His MRI scans are shown in Figures 24c through 24f. Representative histology is shown

in Figures 24g through 24i. The most likely diagnosis is

1- aneurysmal bone cyst.

2- fibrous dysplasia.

3- giant-cell tumor.

4- osteoblastoma.

PREFERRED RESPONSE: 4- osteoblastoma.

DISCUSSION

The combination of immature woven bone with osteoblastic rimming surrounded by a

vascular fibrous bland stroma seen in Figure 24i and scattered giant cells within the lesion

seen in Figure 24h should arouse the suspicion of osteoblastoma. Osteoblastoma most often is

diagnosed in adolescents and young adults, with 75% of patients younger than 25 years of

age at diagnosis. Within tubular bones, osteoblastoma most often arises in the metadiaphyseal

region, but it is more commonly seen in the spine within the posterior elements. The lesion is

well-defined on radiographs, with lucent to mixed lucent and blastic areas. The MRI scan

findings reveal low to intermediate signal intensity on T1- and intermediate to high signal

intensity on T2-weighted images. Neoplastic woven bone can be seen in Figure 24g, and

osteoclasts are shown in Figures 24h and 24i. Although the bone is expansile in appearance

on radiologic studies, there are no large vascular lakes on histology to support the diagnosis

of aneurysmal bone cyst. Although there are areas of woven bone as in fibrous dysplasia, the

appearance of osteoblastic rimming rules against this diagnosis. The location and histology of

the tumor in this case are inconsistent with giant-cell tumor of bone, the latter being a

metaepiphyseal tumor histologically comprising giant cells within a background of stromal

cells having nuclei identical to those of the giant cells.

RECOMMENDED READINGS

Mirra JB. Bone Tumors. Clinical, Radiologic, and Pathologic Correlations. Philadelphia, PA: Lea & Febiger; 1989:391-418.

Nielsen GP, Rosenberg AE. Diagnostic Pathology: Bone. Manitoba, Canada: Amirsys; 2013:1-27.

RESPONSES FOR QUESTIONS 25 THROUGH 29

1- Osteosarcoma

2- Ewing sarcoma

3- Aneurysmal bone cyst

4- Giant-cell tumor

5- Dedifferentiated chondrosarcoma

6- Metastatic disease of bone

Page 17: 2014 Musculoskeletal Tumors and Diseases

7- Multiple myeloma

8- Chordoma

9- Rhabdomyosarcoma

10- Desmoid

11- Parosteal osteosarcoma

Match the condition above with the clinical scenario below.

Question 25 of 143

A 17-year-old girl has knee pain. The imaging shows a distal femur lesion with an adjacent

soft-tissue mass. The cortex is expanded and there is periosteal reaction. There appears to be

a cloud-like density within the lesion.

1- Osteosarcoma

2- Ewing sarcoma

3- Aneurysmal bone cyst

4- Giant-cell tumor

5- Dedifferentiated chondrosarcoma

6- Metastatic disease of bone

7- Multiple myeloma

8- Chordoma

9- Rhabdomyosarcoma

10- Desmoid

11- Parosteal osteosarcoma

PREFERRED RESPONSE: 1- Osteosarcoma

Question 26 of 143

A 44-year-old man has a sacral mass. There is a soft-tissue extension of the mass with bone

involvement centrally within the sacrum. The patient is told that the lesion is radiosensitive.

1- Osteosarcoma

2- Ewing sarcoma

3- Aneurysmal bone cyst

4- Giant-cell tumor

5- Dedifferentiated chondrosarcoma

6- Metastatic disease of bone

7- Multiple myeloma

8- Chordoma

9- Rhabdomyosarcoma

10- Desmoid

11- Parosteal osteosarcoma

PREFERRED RESPONSE: 8- Chordoma

Page 18: 2014 Musculoskeletal Tumors and Diseases

Question 27 of 143

A 13-year-old boy with thigh pain has an enlarging soft-tissue mass. Radiographs show a

mid-diaphyseal lesion with a sunburst pattern of periosteal reaction. A needle biopsy shows

small round blue cells.

1- Osteosarcoma

2- Ewing sarcoma

3- Aneurysmal bone cyst

4- Giant-cell tumor

5- Dedifferentiated chondrosarcoma

6- Metastatic disease of bone

7- Multiple myeloma

8- Chordoma

9- Rhabdomyosarcoma

10- Desmoid

11- Parosteal osteosarcoma

PREFERRED RESPONSE: 2- Ewing sarcoma

Question 28 of 143

An 8-year-old girl has an enlarging mass in her right shoulder. It involves the deltoid and has

a heterogenous pattern on MRI scan. The mass is painless and was noticed when she was

getting bathed 3 months ago. A needle biopsy shows a small round blue-cell tumor.

1- Osteosarcoma

2- Ewing sarcoma

3- Aneurysmal bone cyst

4- Giant-cell tumor

5- Dedifferentiated chondrosarcoma

6- Metastatic disease of bone

7- Multiple myeloma

8- Chordoma

9- Rhabdomyosarcoma

10- Desmoid

11- Parosteal osteosarcoma

PREFERRED RESPONSE: 9- Rhabdomyosarcoma

Question 29 of 143

A 71-year-old man has a history of left hip pain radiating to his knee. The radiographs are

interpreted as osteoarthritis of the hip. He receives a hip injection, but does not experience

relief. A CT scan shows a periacetabular osteolytic lesion with punctuate areas of

calcification and a large soft-tissue mass.

Page 19: 2014 Musculoskeletal Tumors and Diseases

1- Osteosarcoma

2- Ewing sarcoma

3- Aneurysmal bone cyst

4- Giant-cell tumor

5- Dedifferentiated chondrosarcoma

6- Metastatic disease of bone

7- Multiple myeloma

8- Chordoma

9- Rhabdomyosarcoma

10- Desmoid

11- Parosteal osteosarcoma

PREFERRED RESPONSE: 5- Dedifferentiated chondrosarcoma

DISCUSSION

Osteosarcoma is the most common malignant bone tumor and usually involves the

metaphysis of long bones in skeletally immature patients. It is a bone-forming tumor that

often has a cloud-like matrix, which indicates new bone formation. Ewing sarcoma is more

common in the diaphysis. Bone involvement is often accompanied by a soft-tissue mass. It

occurs in a similar age group as osteosarcoma. Ewing sarcoma is radiosensitive; however,

radiation is reserved primarily for nonsurgical anatomic locations. Histologically, it is in the

family of small round blue-cell tumors. Rhabdomyosarcoma is a high-grade soft-tissue

sarcoma most commonly seen in children. Like most soft-tissue sarcomas, it often presents as

an enlarging soft-tissue mass. It is treated with chemotherapy and surgical resection.

Radiation may also be used to reduce risk for local recurrence. Rhabdomyosarcoma is in the

family of small round blue-cell tumors. Chordoma occurs in 2 locations; the central portion

of the lower spine, usually the sacrum, and the base of the skull (the clivus). It is a

radiosensitive tumor; however, resection with a negative margin is the recommended

treatment. Dedifferentiated chondrosarcoma is a high-grade malignant tumor. Histologically,

areas consistent with both chondrosarcoma and osteosarcoma are often found. Other high-

grade tumors have also been described in combination with chondrosarcoma, such as

malignant fibrous histiocytoma of bone. In the pelvis, it can be difficult to see on radiographs,

and the symptoms can be confused with more common entities such as osteoarthritis or Paget

disease. The prognosis for these patients is poor, and chemotherapy use is controversial.

RECOMMENDED READINGS

Lewis VO, Morris CD, Parsons TW. Malignant and benign bone tumors that you are likely to see. Instr Course Lect. 2013;62:535-49. PubMed PMID: 23395056.

View Abstract at PubMed

Schwab JH, Springfield DS, Raskin KA, Mankin HJ, Hornicek FJ. What's new in primary bone tumors. J Bone Joint Surg Am. 2012 Oct

17;94(20):1913-9. doi: 10.2106/JBJS.L.00955. Review. PubMed PMID: 23079883.

View Abstract at PubMed

Peabody TD, Gibbs CP Jr, Simon MA. Evaluation and staging of musculoskeletal neoplasms. J Bone Joint Surg Am. 1998 Aug;80(8):1204-

18. Review. PubMed PMID: 9730132.

View Abstract at PubMed

Page 20: 2014 Musculoskeletal Tumors and Diseases

Figure 30a

Figure 30b

Figure 30c

Question 30 of 143

Figure 30a is the radiograph taken in the emergency department of a 20-year-old man with

pain and swelling in his right-dominant arm. His MRI scan is seen in Figure 30b, and his

histopathology is shown in Figure 30c. What is the most likely diagnosis?

1- Ewing sarcoma

2- Langerhans cell histiocytosis

3- Osteosarcoma

4- Osteomyelitis

PREFERRED RESPONSE: 1- Ewing sarcoma

DISCUSSION

The imaging shows a permeative lytic diaphyseal lesion with malignant periosteal reaction

and a large soft-tissue mass. The histology shows a small round blue-cell tumor consistent

with Ewing sarcoma or lymphoma; genetic or immunohistochemical studies can be used for

the final differentiation. Langerhans cell histiocytosis or osteomyelitis would not present this

histology. Osteosarcoma would more likely be metaphyseal and would show osteoid

production on radiographs and histology.

RECOMMENDED READINGS

Peersman B, Vanhoenacker FM, Heyman S, Van Herendael B, Stam M, Brys P, Verstraete KL, Samson I, Sybers J, Van Dyck P, Parizel PM, De Schepper AM. Ewing's sarcoma: imaging features. JBR-BTR. 2007 Sep-Oct;90(5):368-76. PubMed PMID: 18085191.

View Abstract at PubMed

Bacci G, Balladelli A, Forni C, Longhi A, Serra M, Fabbri N, Alberghini M, Ferrari S, Benassi MS, Picci P. Ewing's sarcoma family

tumours. Differences in clinicopathological characteristics at presentation between localised and metastatic tumours. J Bone Joint Surg Br.

2007 Sep;89(9):1229-33. PubMed PMID: 17905963. View Abstract at PubMed

Page 21: 2014 Musculoskeletal Tumors and Diseases

Arndt CA, Rose PS, Folpe AL, Laack NN. Common musculoskeletal tumors of childhood and adolescence. Mayo Clin Proc. 2012

May;87(5):475-87. doi: 10.1016/j.mayocp.2012.01.015. Review. PubMed PMID: 22560526. View Abstract at PubMed

Figure 31a

Figure 31b

Figure 31c

Figure 31d

Question 31 of 143

An 18-year-old woman has had a right thigh mass for “a long time.” She has found it

increasingly difficult to ambulate. Her medical history is unremarkable, with the exception of

hypertension. On clinical examination, a fullness is appreciated in the popliteal fossa. The

neurovascular status of the distal extremity is intact. Anteroposterior and lateral radiographs

are shown in Figures 31a and 31b. A lateral T2-weighted MRI scan is shown in Figure 31c.

The histology is seen in Figure 31d. What is the most likely diagnosis?

1- Aneurysmal bone cyst

2- Osteosarcoma

3- Periosteal lipoma

4- Parosteal osteosarcoma

PREFERRED RESPONSE: 2- Osteosarcoma

DISCUSSION

The radiographs indicate a bone-forming tumor that has intramedullary and extramedullary

components. The MRI scan confirms the extent of the tumor. Parosteal osteosarcomas are

generally surface tumors sparing the medullary canal, although extension into the canal can

occur. However, the histology is low-grade with a well-differentiated fibrous stroma, unlike

this case. The histology here shows pleomorphic cells, with the haphazard tumor bone

deposition consistent with a high-grade conventional osteosarcoma. Telangiectatic

osteosarcoma is associated with vast lakes of blood, which are usually seen on MRI scan as

fluid-fluid levels. In this patient, there was a single fluid-fluid level on MRI scan, but the

histology provided did not show lakes of blood. Aneurysmal bone cyst and periosteal lipoma

are both benign lesions and not consistent with the histology shown.

RECOMMENDED READINGS

Aboulafia AJ, Malawer MM. Surgical management of pelvic and extremity osteosarcoma. Cancer. 1993 May 15;71(10 Suppl):3358-66. Review. PubMed PMID: 8490883.

View Abstract at PubMed

Page 22: 2014 Musculoskeletal Tumors and Diseases

Messerschmitt PJ, Garcia RM, Abdul-Karim FW, Greenfield EM, Getty PJ. Osteosarcoma. J Am Acad Orthop Surg. 2009 Aug;17(8):515-

27. Review. PubMed PMID: 19652033. View Abstract at PubMed

Figure 32a

Figure 32b

CLINICAL SITUATION FOR QUESTIONS 32 THROUGH 36

A 68-year-old woman is referred for left thigh pain. Her medical history includes

hypertension, diabetes, and adenocarcinoma of the breast treated with surgery, chemotherapy,

and radiation 3 years ago. She currently is on aromatase therapy. She is unable to ambulate

secondary to pain, is limited to a walker, and requires narcotic medications. She has no other

pain but agrees to your recommendation that she urgently be sent to the hospital.

Question 32 of 143

After viewing the radiographs in Figures 32a and 32b, what is the most appropriate next step?

1- Surgical stabilization

2- Biopsy of the lesion

3- Further imaging

4- Immediate administration of bisphosphonates

PREFERRED RESPONSE: 3- Further imaging

Question 33 of 143

The patient is scheduled for surgery to stabilize the bone. According to the Mirels Scoring

System, what is most likely related to risk for pathologic fracture?

1- Pathology of the original tumor

2- Gender

3- Location of the lesion

4- Age of the patient

PREFERRED RESPONSE: 3- Location of the lesion

Page 23: 2014 Musculoskeletal Tumors and Diseases

Question 34 of 143

The surgeon decides to obtain a biopsy before stabilization; new lesions are unrelated to

previous malignancies in what percentage of cases?

1- 5%

2- 15%

3- 25%

4- 35%

PREFERRED RESPONSE: 2- 15%

Question 35 of 143

Chance for fracture healing often is a factor in surgical decision making for reconstruction of

pathologic lesions. Among the diagnoses listed below, which grouping represents fracture

healing likelihood from least likely to most likely?

1- Breast carcinoma, lung carcinoma, multiple myeloma,

2- Multiple myeloma, breast carcinoma, lung carcinoma

3- Lung carcinoma, multiple myeloma, breast carcinoma

4- Lung carcinoma, breast carcinoma, multiple myeloma

PREFERRED RESPONSE: 4- Lung carcinoma, breast carcinoma, multiple myeloma

Question 36 of 143

Which intervention should be added to this patient’s care to best prevent future skeletally

related events (SRE)?

1- Inferior vena cava (IVC) filter placement

2- Bisphosphonates

3- External beam radiation

4- Tc-99 whole-body bone scan at regular intervals

PREFERRED RESPONSE: 2- Bisphosphonates

DISCUSSION

A new lesion in a patient with known carcinoma but without bone disease needs to be

evaluated before stabilization. As shown by Clayer and associates, as many as 15% of new

bone lesions with a history of known carcinoma will be from a different primary and should

be worked up before surgery. Evaluation with appropriate imaging will allow for

identification of other lesions that may be easier to biopsy or require fixation. Mirels

classified the chance that a metastatic lesion in the extremities will go on to pathologic

fracture based on 4 factors: size, site, type of bone reaction, and pain level. Prophylactic

Page 24: 2014 Musculoskeletal Tumors and Diseases

fixation is recommended for scores of 8 or 9 and above, with a 15% to 30% chance of

fracture, respectively. The pathologic diagnosis is not involved with the chance for fracture

based on the Mirels classification.

Pathologic diagnoses, however, are associated with healing likelihood of lesions and the time

at which replacement should be considered over reduction and internal fixation. While not

technically metastatic disease, the systemic nature of multiple myeloma functions like a

widespread disease. Lung carcinoma has a very low healing likelihood, followed by breast

cancer, renal cancer, and myeloma, and should be factored when choosing methods of

surgical stabilization as shown by Gainor and associates.

Finally, because these patients are at significant risk for more lesions and skeletal-related

events, appropriate systemic treatment with bisphosphonates has been shown to help with

hypercalcemia and reduce SREs. IVC filter placement has been advocated to reduce risk for

fatal pulmonary embolism in patients with lower-extremity metastatic disease undergoing

operative intervention. Radiation is indicated for local control, and monitoring with bone

scans will help identify impending lesions sooner than relying on pain symptoms alone.

RECOMMENDED READINGS

Bickels J, Dadia S, Lidar Z. Surgical management of metastatic bone disease. J Bone Joint Surg Am. 2009 Jun;91(6):1503-16. doi:

10.2106/JBJS.H.00175. Review. PubMed PMID: 19487532.

View Abstract at PubMed

Clayer M, Duncan W. Importance of biopsy of new bone lesions in patients with previous carcinoma. Clin Orthop Relat Res. 2006 Oct;451:208-11. PubMed PMID: 16801861.

View Abstract at PubMed

Gainor BJ, Buchert P. Fracture healing in metastatic bone disease. Clin Orthop Relat Res. 1983 Sep;(178):297-302. PubMed PMID:

6883864.

View Abstract at PubMed

Mirels H. Metastatic disease in long bones. A proposed scoring system for diagnosing impending pathologic fractures. Clin Orthop Relat

Res. 1989 Dec;(249):256-64. PubMed PMID: 2684463. View Abstract at PubMed

Quinn RH. Surgical management of lower extremity metastatic disease. In: Schwartz H, ed. Orthopaedic Knowledge Update: Musculoskeletal Tumors 2. Rosemont, IL: American Academy of Orthopaedic Surgeons; 2007:383-392.

Figure 37a

Figure 37b

Figure 37c

Question 37 of 143

Page 25: 2014 Musculoskeletal Tumors and Diseases

Figures 37a and 37b are the lateral and anteroposterior (AP) radiographs of a 60-year-old

man with a remote history of renal cell cancer. A needle biopsy of the lesion is shown in

Figure 37c. The bone destruction that occurs in this process is a result of

1- tumor cells.

2- cytokines secreted by the tumor.

3- host bone osteoblasts.

4- osteoprotegerin.

PREFERRED RESPONSE: 2- cytokines secreted by the tumor.

DISCUSSION

The lateral and AP radiographs of the elbow reveal a lucent lesion consistent with metastatic

disease, and, specifically, metastatic renal cancer. The histology is consistent with clear cell

carcinoma. Tumor cells do not destroy host bone. Rather, cytokines from the tumor stimulate

osteoclasts or osteoblasts to destroy or generate new bone, respectively. Osteoblasts secrete

receptor activator of nuclear factor kappa beta ligand (RANKL), and osteoclasts have

receptors for RANKL. This causes an increase in osteoclast precursors, resulting in increased

bone destruction. Osteoprotegerin is a decoy receptor that binds to RANKL and inhibits an

increase in osteoclasts.

RECOMMENDED READINGS

Mundy GR. Metastasis to bone: causes, consequences and therapeutic opportunities. Nat Rev Cancer. 2002 Aug;2(8):584-93. Review.

PubMed PMID: 12154351.

View Abstract at PubMed

Roodman GD. Mechanisms of bone metastasis. N Engl J Med. 2004 Apr 15;350(16):1655-64. Review. PubMed PMID: 15084698.

View Abstract at PubMed

Figure 38a

Figure 38b

Question 38 of 143

Figures 38a and 38b are the histopathology of an otherwise healthy 31-year-old man who had

a growing mass excised from his forearm with local anesthetic and no preoperative imaging.

The mass was documented to be subfascial and larger than 5 cm. What is the best local

treatment option?

1- Observation

2- Radiation only

3- Chemotherapy only

4- Re-excision and radiation

PREFERRED RESPONSE: 4- Re-excision and radiation

Page 26: 2014 Musculoskeletal Tumors and Diseases

DISCUSSION

The misdiagnosis of a soft-tissue mass is fairly common. However, in this case the first

indication that the presumed diagnosis is incorrect is supported by significant growth over a

short time period. Generally, lipomas and many other benign soft-tissue masses grow slowly

over years rather than weeks or months. On MRI scan, a lipoma would classically

demonstrate homogenous high-signal intensity on a T1 sequence and fairly low-signal

intensity on a T2 sequence. No imaging was done before surgery, but a high-grade sarcoma

generally shows a heterogenous signal on T1 and T2 sequences that is distinctly different

than the pattern of a lipoma. The histopathology is consistent with a high-grade sarcoma A

spindle-cell tumor with hypercellularity, anaplasia, and pleomorphism. Mitotic figures are

also noted, suggesting rapid turnover and growth. This is a high-grade sarcoma that is treated

with wide surgical excision and observation is not recommended. External beam radiation is

often used as an adjuvant either before or after surgery to help decrease the local recurrence

rate. Radiation alone would not be sufficient treatment. The use of chemotherapy is

controversial. In the setting of a “whoops” or unintended resection of a high-grade soft-tissue

sarcoma, re-excision of the tumor bed with radiation is generally indicated. The radiation is

used to eliminate microscopic contamination. However, there is some evidence to support

surgery alone for superficial (subcutaneous) sarcomas.

RECOMMENDED READINGS

Mankin HJ, Mankin CJ, Simon MA. The hazards of the biopsy, revisited. Members of the Musculoskeletal Tumor Society. J Bone Joint

Surg Am. 1996 May;78(5):656-63. PubMed PMID: 8642021.

View Abstract at PubMed

Noria S, Davis A, Kandel R, Levesque J, O'Sullivan B, Wunder J, Bell R. Residual disease following unplanned excision of soft-tissue

sarcoma of an extremity. J Bone Joint Surg Am. 1996 May;78(5):650-5. PubMed PMID: 8642020. View Abstract at PubMed

Gibbs CP, Peabody TD, Mundt AJ, Montag AG, Simon MA. Oncological outcomes of operative treatment of subcutaneous soft-tissue sarcomas of the extremities. J Bone Joint Surg Am. 1997 Jun;79(6):888-97. PubMed PMID: 9199387.

View Abstract at PubMed

Figure 39a

Figure 39b

Figure 39c

Figure 39d

Figure 39e

Question 39 of 143

Figure 39a is the anteroposterior radiograph of a 51-year-old man who has had a painless

soft-tissue mass on his left wrist for 2 months. MR sequences are shown in Figures 39b

through 39d. A biopsy was performed and shown in a low-power hematoxylin and eosin

photomicrograph in Figure 39e. The most appropriate treatment for this lesion is

Page 27: 2014 Musculoskeletal Tumors and Diseases

1- a diet that reduces uric acid production.

2- wide local resection followed by radiotherapy.

3- marginal excision.

4- observation until the mineralization matures, and then excision and radiotherapy to prevent

recurrence.

PREFERRED RESPONSE: 3- marginal excision.

DISCUSSION

The tumor seen here is a giant-cell tumor of the tendon sheath. A solid lesion larger than 1.5

cm in the wrist warrants biopsy to exclude malignancy. Synovial sarcoma should be

considered in mineralized periarticular masses; histology does not support this diagnosis. The

3 most common benign tumors of the wrist are ganglion cysts, giant-cell tumors of tendon

sheath, and hemangiomas. Radiographs are nonspecific but may reveal extrinsic erosions of

bone by giant-cell tumor of tendon sheath. MR findings of prominent low-signal intensity

(seen with T2-weighting) and “blooming” artifact from the hemosiderin (seen with gradient-

echo sequences) are nearly pathognomonic of this diagnosis as seen here. Pathology shows

hemosiderin-laden macrophages and giant cells as seen in the histology. Uric acid reduction

would be indicated for gout and wide local excision for a soft-tissue sarcoma. Observation

awaiting mature mineralization would apply to heterotopic ossification.

RECOMMENDED READINGS

Nguyen V, Choi J, Davis KW. Imaging of wrist masses. Curr Probl Diagn Radiol. 2004 Jul-Aug;33(4):147-60. Review. PubMed PMID: 15306759.

View Abstract at PubMed

Murphey MD, Rhee JH, Lewis RB, Fanburg-Smith JC, Flemming DJ, Walker EA. Pigmented villonodular synovitis: radiologic-pathologic

correlation. Radiographics. 2008 Sep-Oct;28(5):1493-518. doi: 10.1148/rg.285085134. PubMed PMID: 18794322.

View Abstract at PubMed

Figure 40a

Figure 40b

Figure 40c

CLINICAL SITUATION FOR QUESTIONS 40 THROUGH 43

Page 28: 2014 Musculoskeletal Tumors and Diseases

Figures 40a through 40c are the radiograph, bone scan, and histology of a 68-year-old man

who has had 3 months of pain in his left thigh with weight bearing. He has no history of

cancer and no illnesses.

Question 40 of 143

The lesion seen in Figures 40a and 40b may best be described as

1- malignant.

2- benign latent.

3- benign aggressive.

4- inflammatory.

PREFERRED RESPONSE: 1- malignant.

Question 41 of 143

The origin of the lesion seen in Figures 40a and 40b may best be determined with

1- CT scan of the chest, abdomen and pelvis.

2- CT scan of the lesion.

3- MRI scan.

4- I-131 scan.

PREFERRED RESPONSE: 1- CT scan of the chest, abdomen and pelvis.

Question 42 of 143

The best next diagnostic step after imaging and staging is

1- prophylactic nail with reamings sent for permanent pathology.

2- biopsy.

3- C-reactive protein.

4- MRI scan.

PREFERRED RESPONSE: 2- biopsy.

Question 43 of 143

The orthopaedic surgeon obtains tissue with the histology shown in Figure 40c. Treatment

should consist of

1- surgical stabilization.

2- surgical stabilization and radiation.

Page 29: 2014 Musculoskeletal Tumors and Diseases

3- excision and endoprosthesis.

4- radiation.

PREFERRED RESPONSE: 2- surgical stabilization and radiation.

DISCUSSION

The history and images show a lytic, malignant solitary bone lesion with a pathologic fracture

in an adult patient. In a patient older than age 50 with a lytic bone lesion, metastatic disease is

the most likely diagnosis. Prostate, thyroid, breast, lung, and kidney are the most common

cancer sources for a primary lesion. The most likely test that will identify the primary

malignancy is a CT scan of the chest, abdomen, and pelvis. Because no previous cancer

history exists, a biopsy of the bone lesion is necessary to make a diagnosis and to rule out a

primary bone sarcoma, for which the treatment would vastly differ. In a patient with no

cancer history, the biopsy is obtained to exclude a primary bone malignancy before surgical

intervention. The histology in this case shows nests of cohesive cells mimicking glandular

structures consistent with a carcinoma, so surgical stabilization may be undertaken.

Cementation and curettage alone will not address the mechanical instability, excision and

endoprosthesis would entail a larger surgery than is needed, and radiation alone will not

address the mechanical instability. In the setting of placing an intramedullary nail, the bone

will require radiation to eliminate residual disease within the femur.

RECOMMENDED READINGS

Weber KL. Evaluation of the adult patient (aged >40 years) with a destructive bone lesion. J Am Acad Orthop Surg. 2010 Mar;18(3):169-79. Review. PubMed PMID: 20190107.

View Abstract at PubMed

Cronin CG, Cashell T, Mhuircheartaigh JN, Swords R, Murray M, O'Sullivan GJ, O'Keeffe D. Bone biopsy of new suspicious bone lesions

in patients with primary carcinoma: prevalence and probability of an alternative diagnosis. AJR Am J Roentgenol. 2009 Nov;193(5):W407-

10. doi: 10.2214/AJR.08.1882. PubMed PMID: 19843719. View Abstract at PubMed

Weber KL, Peabody T, Frassica FJ, Mott MP, Parsons TW 3rd. Tumors for the general orthopedist: how to save your patients and practice. Instr Course Lect. 2010;59:579-91. PubMed PMID: 20415407.

View Abstract at PubMed

Figure 44a

Figure 44b

Question 44 of 143

Page 30: 2014 Musculoskeletal Tumors and Diseases

Figures 44a and 44b are the anteroposterior hand and humerus radiographs of a 5-year-old

girl who has painful masses in her hand. Her parents noted concern about her left wrist and

upper arm, which have been sites of occasional pain. Examination reveals obvious deformity

of the left upper arm with shortening, mild bowing, and nonmobile masses within each

finger. No evidence of lesions is seen on her skin or in the soft tissues. What is the

approximate lifetime incidence of malignant degeneration from this condition?

1- 1% to 5%

2- 10% to 25%

3- 50% to 70%

4- 80% to 100%

PREFERRED RESPONSE: 2- 10% to 25%

DISCUSSION

This is a good example of a patient with Ollier disease, or multiple enchondromatosis. The

lack of skin or soft-tissue findings suggests there are no signs of angiomas/hemangiomas that

are required to make the diagnosis of Maffucci syndrome. Ollier disease is associated with

malignant degeneration of benign cartilaginous tumors into chondrosarcomas with variable

reported rates. The reported incidence ranges from 10% to 25%. It is suggested that patients

with Maffucci syndrome have cancer rates approaching 100%; however, in many of these

cases the malignancy is not from sarcomatous transformation of an enchondroma, but rather

other primary sources such as the gastrointestinal tract.

RECOMMENDED READINGS

Ghatan A, Scharschmidt T, Conrad E. Extreme enchondromatosis: a report of two cases and review of the literature. J Bone Joint Surg Am.

2010 Oct 6;92(13):2336-43. doi: 10.2106/JBJS.J.00194. Review. PubMed PMID: 20926729.

View Abstract at PubMed

Pansuriya TC, Kroon HM, Bovée JV. Enchondromatosis: insights on the different subtypes. Int J Clin Exp Pathol. 2010 Jun 26;3(6):557-69.

Review. PubMed PMID: 20661403. View Abstract at PubMed

Figure 45a

Figure 45b

Figure 45c

CLINICAL SITUATION FOR QUESTIONS 45 AND 46

Figures 45a through 45c show the sagittal proton density, T2-weighted, and axial gradient

echo MRI scans of a 34-year-old active woman with knee pain and swelling. Radiographs

show only an effusion.

Question 45 of 143

Page 31: 2014 Musculoskeletal Tumors and Diseases

The most likely diagnosis is

1- synovial sarcoma.

2- synovial chondromatosis.

3- pigmented villonodular synovitis (PVNS).

4- posttraumatic hemarthrosis.

PREFERRED RESPONSE: 3- pigmented villonodular synovitis (PVNS).

Question 46 of 143

First-line treatment recommendations include

1- synovectomy.

2- arthrocentesis, compressive wrap, and rest.

3- en bloc resection.

4- intra-articular radioactive nucleotide injection.

PREFERRED RESPONSE: 1- synovectomy.

DISCUSSION

The MRI scans reveal classic findings of PVNS with low signal on proton density and T2

sequences and evidence of “blooming” on gradient echo sequences. A discrete mass

suggestive of a sarcoma (which would typically appear dark on T1 and bright on T2, but

heterogenous and enhancing) is not seen, and synovial sarcomas rarely arise in an intra-

articular location. Cartilaginous nodules of synovial chondromatosis are not seen, and the

abnormal synovial process indicates that this is more than just a posttraumatic hemarthrosis.

Treatment is synovectomy; radiotherapy is reserved for select recurrent cases. Arthrocentesis

would be used for a simple hemarthrosis, and resection used for a sarcoma.

RECOMMENDED READINGS

Tyler WK, Vidal AF, Williams RJ, Healey JH. Pigmented villonodular synovitis. J Am Acad Orthop Surg. 2006 Jun;14(6):376-85. Review. PubMed PMID: 16757677.

View Abstract at PubMed

Colman MW, Ye J, Weiss KR, Goodman MA, McGough RL 3rd. Does combined open and arthroscopic synovectomy for diffuse PVNS of

the knee improve recurrence rates? Clin Orthop Relat Res. 2013 Mar;471(3):883-90. doi: 10.1007/s11999-012-2589-8. PubMed PMID:

22996360. View Abstract at PubMed

Ogilvie-Harris DJ, McLean J, Zarnett ME. Pigmented villonodular synovitis of the knee. The results of total arthroscopic synovectomy, partial, arthroscopic synovectomy, and arthroscopic local excision. J Bone Joint Surg Am. 1992 Jan;74(1):119-23. PubMed PMID: 1463472.

View Abstract at PubMed

RESPONSES FOR QUESTIONS 47 THROUGH 52

1- Ultrasound

2- MRI scan of the thigh

Page 32: 2014 Musculoskeletal Tumors and Diseases

3- Chest CT scan and whole-body bone scan

4- Positron emission tomography (PET) scan

5- Presurgical radiation therapy

6- Marginal resection

7- Radical resection and postsurgical radiation

8- Transverse incision centered over the mass

9- Incision centered over the mass in line with long axis of limb

10- Sentinel node biopsy

11- Core needle biopsy

For each soft-tissue mass clinical scenario or question below, match the most

appropriate next evaluation or treatment step listed above.

Question 47 of 143

A 34-year-old man with a large and rapidly enlarging deep mass in his thigh.

1- Ultrasound

2- MRI scan of the thigh

3- Chest CT scan and whole-body bone scan

4- Positron emission tomography (PET) scan

5- Presurgical radiation therapy

6- Marginal resection

7- Radical resection and postsurgical radiation

8- Transverse incision centered over the mass

9- Incision centered over the mass in line with long axis of limb

10- Sentinel node biopsy

11- Core needle biopsy

PREFERRED RESPONSE: 2- MRI scan of the thigh

Question 48 of 143

A 56-year-old woman with a heterogenous retroperitoneal soft-tissue mass larger than 5 cm

(as imaged on MRI scan) that first was noticed after a slip and fall. Staging study findings are

negative.

1- Ultrasound

2- MRI scan of the thigh

3- Chest CT scan and whole-body bone scan

4- Positron emission tomography (PET) scan

5- Presurgical radiation therapy

6- Marginal resection

7- Radical resection and postsurgical radiation

8- Transverse incision centered over the mass

9- Incision centered over the mass in line with long axis of limb

10- Sentinel node biopsy

11- Core needle biopsy

Page 33: 2014 Musculoskeletal Tumors and Diseases

PREFERRED RESPONSE: 11- Core needle biopsy

Question 49 of 143

A 65-year-old man with an American Joint Committee on Cancer (AJCC) III anterior thigh

sarcoma is considering treatment options. What treatment regimen is associated with a higher

rate of wound healing complications after surgery?

1- Ultrasound

2- MRI scan of the thigh

3- Chest CT scan and whole-body bone scan

4- Positron emission tomography (PET) scan

5- Presurgical radiation therapy

6- Marginal resection

7- Radical resection and postsurgical radiation

8- Transverse incision centered over the mass

9- Incision centered over the mass in line with long axis of limb

10- Sentinel node biopsy

11- Core needle biopsy

PREFERRED RESPONSE: 5- Presurgical radiation therapy

Question 50 of 143

A 44-year-old woman elects surgical treatment of an intramuscular lipoma. What is the

preferred surgical treatment?

1- Ultrasound

2- MRI scan of the thigh

3- Chest CT scan and whole-body bone scan

4- Positron emission tomography (PET) scan

5- Presurgical radiation therapy

6- Marginal resection

7- Radical resection and postsurgical radiation

8- Transverse incision centered over the mass

9- Incision centered over the mass in line with long axis of limb

10- Sentinel node biopsy

11- Core needle biopsy

PREFERRED RESPONSE: 6- Marginal resection

Question 51 of 143

Page 34: 2014 Musculoskeletal Tumors and Diseases

A 29-year-old man with a large and enlarging upper arm deep mass has an inconclusive

needle biopsy.

1- Ultrasound

2- MRI scan of the thigh

3- Chest CT scan and whole-body bone scan

4- Positron emission tomography (PET) scan

5- Presurgical radiation therapy

6- Marginal resection

7- Radical resection and postsurgical radiation

8- Transverse incision centered over the mass

9- Incision centered over the mass in line with long axis of limb

10- Sentinel node biopsy

11- Core needle biopsy

PREFERRED RESPONSE: 9- Incision centered over the mass in line with long axis of limb

Question 52 of 143

A 60-year-old woman with a clear cell sarcoma of the foot has no evidence of disease

elsewhere after standard staging of a soft-tissue sarcoma. What additional evaluation is

recommended?

1- Ultrasound

2- MRI scan of the thigh

3- Chest CT scan and whole-body bone scan

4- Positron emission tomography (PET) scan

5- Presurgical radiation therapy

6- Marginal resection

7- Radical resection and postsurgical radiation

8- Transverse incision centered over the mass

9- Incision centered over the mass in line with long axis of limb

10- Sentinel node biopsy

11- Core needle biopsy

PREFERRED RESPONSE: 10- Sentinel node biopsy

DISCUSSION

In patients with rapidly enlarging painless masses, particularly those that are either large or

deep, the diagnosis of a soft-tissue sarcoma should be entertained. Characteristics that are

concerning are masses larger than 5 cm in largest dimension that are subfascial and

heterogenous on MRI scan. MRI scan is the preferred imaging modality for evaluation of

soft-tissue sarcomas because it can delineate location of the lesion, involvement of

neurovascular structures, intra-articular involvement, and underlying signal alteration in the

osseous structures. Most patients relate the onset of their symptoms to a traumatic event, and

the interpretation of the MRI scan can include a hematoma. Ultrasound is more commonly

used to confirm the clinical impression of a cyst and to distinguish cystic from solid masses.

Page 35: 2014 Musculoskeletal Tumors and Diseases

A core needle biopsy can easily be performed in an outpatient setting or with image guidance

to aid in the diagnosis. Core needle biopsies preserve the architectural relationship of cells,

which is important in the diagnosis of mesenchymal lesions. A fine-needle aspiration allows

cytologic but not histologic analysis, and generally is not favored for the diagnosis of a

sarcoma. If an open biopsy is performed, the incision must be oriented in line with the long

axis of the extremity to minimize contamination of surrounding structures. Meticulous

hemostasis and closure of the tumor pseudocapsule must be achieved to minimize local tissue

contamination. An open biopsy is commonly used if an initial needle biopsy is nondiagnostic.

Transverse biopsy incisions should not be used.

Presurgical radiation for sarcomas is advantageous to decrease the field of radiation and

overall radiation dose, but a significant postsurgical wound-healing complication rate (up to

35%) is associated with presurgical radiation therapy.

Staging of sarcomas is important to predict prognosis. Not all sarcomas are reliably positive

on a PET scan, so the preferred staging studies are CT scan of the chest and whole-body bone

scan. Hematogenous spread of sarcomas is the most common route of metastatic disease,

which speaks to the value of chest CT scans. Clear cell sarcomas (in addition to synovial

sarcoma, angiosarcoma, epitheliod sarcoma, and rhabdomyosarcoma) have a propensity for

lymphatic nodal metastatic disease, so sentinel node biopsy is considered in assessing these

tumors. Recent evidence supports efficacy of sentinel node biopsy for clear cell sarcomas in

particular.

RECOMMENDED READINGS

Edge SB, Byrd DR, Compton CC, Fritz AG, Greene FL, Trotti A, eds. AJCC Cancer Staging Manual. 7th ed. New York, NY: Springer;

2010.

Arbeit JM, Hilaris BS, Brennan MF. Wound complications in the multimodality treatment of extremity and superficial truncal sarcomas. J Clin Oncol. 1987 Mar;5(3):480-8. PubMed PMID: 3819810.

View Abstract at PubMed

Luce EA. The irradiated wound. Surg Clin North Am. 1984 Aug;64(4):821-9. PubMed PMID: 6591499.

View Abstract at PubMed

Cheng EY, Dusenbery KE, Winters MR, Thompson RC. Soft tissue sarcomas: preoperative versus postoperative radiotherapy. J Surg Oncol.

1996 Feb;61(2):90-9. PubMed PMID: 8606553.

View Abstract at PubMed

Mankin HJ, Lange TA, Spanier SS. The hazards of biopsy in patients with malignant primary bone and soft-tissue tumors. J Bone Joint Surg

Am. 1982 Oct;64(8):1121-7. PubMed PMID: 7130225. View Abstract at PubMed

Singer S, Nielsen T, Antonescu CR. Molecular biology of soft tissue sarcoma. In: DeVita VT Jr, Lawrence TS, Rosenberg SA, eds. Cancer: Principles and Practice of Oncology. 9th ed. Philadelphia, PA: Lippincott Williams & Wilkins; 2011:1522-1532.

Singer S, Maki RG, O'Sullivan B. Soft tissue sarcoma. In: DeVita VT Jr, Lawrence TS, Rosenberg SA, eds. Cancer: Principles and Practice

of Oncology. 9th ed. Philadelphia, PA: Lippincott Williams & Wilkins; 2011:1533-1577.

Malawer MM, Helman LJ, O'Sullivan B. Sarcomas of bone. In: DeVita VT Jr, Lawrence TS, Rosenberg SA, eds. Cancer: Principles and

Practice of Oncology. 9th ed. Philadelphia, PA: Lippincott Williams & Wilkins; 2011:1578-1609.

O'Sullivan B, Davis AM, Turcotte R, Bell R, Catton C, Chabot P, Wunder J, Kandel R, Goddard K, Sadura A, Pater J, Zee B. Preoperative versus postoperative radiotherapy in soft-tissue sarcoma of the limbs: a randomised trial. Lancet. 2002 Jun 29;359(9325):2235-41. PubMed

Page 36: 2014 Musculoskeletal Tumors and Diseases

PMID: 12103287.

View Abstract at PubMed

Andreou D, Boldt H, Werner M, Hamann C, Pink D, Tunn PU. Sentinel node biopsy in soft tissue sarcoma subtypes with a high propensity

for regional lymphatic spread--results of a large prospective trial. Ann Oncol. 2013 May;24(5):1400-5. doi: 10.1093/annonc/mds650. Epub 2013 Jan 31. PubMed PMID: 23372051.

View Abstract at PubMed

Figure 53

Question 53 of 143

Figure 53 is the emergency department radiograph of a 7-year-old boy who has pain and is

unwilling to use his right arm after a fall on the playground. What is the most appropriate

initial treatment?

1- Nonsurgical treatment of the fracture

2- Aspiration and injection with methylprednisolone

3- Curettage and augmentation with bone cement and internal fixation

4- Further imaging and biopsy

PREFERRED RESPONSE: 1- Nonsurgical treatment of the fracture

DISCUSSION

When a unicameral cyst is present in the setting of a pathological fracture, it is best to allow

the fracture to heal before administering interventions for the cyst. In some cases, waiting

may obviate the need for a procedure to manage the cyst. When indicated, the initial

procedure of choice is aspiration and injection of the cyst. Although many variations of the

procedure exist, current evidence continues to support the role of methylprednisolone in the

injection portion of the procedure. Curettage and packing may sometimes be appropriate for

recalcitrant cysts, but is not indicated for this patient. These cysts are typically solitary and

are not considered at risk for malignant degeneration or systemic involvement; therefore,

further distant imaging and biopsy is unnecessary.

RECOMMENDED READINGS

Baig R, Eady JL. Unicameral (simple) bone cysts. South Med J. 2006 Sep;99(9):966-76. Review. PubMed PMID: 17004531.

View Abstract at PubMed

Donaldson S, Wright JG. Recent developments in treatment for simple bone cysts. Curr Opin Pediatr. 2011 Feb;23(1):73-7. doi:

10.1097/MOP.0b013e3283421111. Review. PubMed PMID: 21191299. View Abstract at PubMed

Lokiec F, Wientroub S. Simple bone cyst: etiology, classification, pathology, and treatment modalities. J Pediatr Orthop B. 1998 Oct;7(4):262-73. Review. PubMed PMID: 9810525.

View Abstract at PubMed

Page 37: 2014 Musculoskeletal Tumors and Diseases

Figure 54

Question 54 of 143

Figure 54 is the CT scan of a 70-year-old man with progressive neck pain; there is no history

of trauma, and examination is notable only for mildly decreased cervical range of motion. He

is neurologically intact. He has monoclonal gammopathy of undetermined significance that

has been stable for many years. Current serum protein electrophoresis is unchanged. History

and examination reveal no other causes for his pain. What is the next step in clinical

management?

1- Corpectomy and anterior fusion

2- Radiation therapy followed by multiple myeloma protocol chemotherapy

3- CT-guided biopsy

4- CT scan of the chest, abdomen, and pelvis

PREFERRED RESPONSE: 4- CT scan of the chest, abdomen, and pelvis

DISCUSSION

The patient has a lytic lesion of unclear etiology. Rougraff and associates demonstrated the

efficacy of a protocol including history and examination, basic laboratory studies, imaging,

and biopsy of the most accessible lesion in identifying the primary site of disease in patients

with skeletal metastases of unknown primary. A CT scan of the chest, abdomen, and pelvis

and bone scan complete the next step of Rougraff’s protocol. Although the history suggests

possible myeloma, the laboratory studies do not support transformation of monoclonal

gammopathy to multiple myeloma. For this patient, CT scan showed a primary lung cancer,

and a biopsy of a rib metastasis (much safer to biopsy than the cervical spine) confirmed

metastatic adenocarcinoma.

Reflexive biopsy of a difficult and risky lesion is incorrect; staging the patient is likely to

identify a safer site to access for biopsy and may identify conditions such as metastatic renal

cell carcinoma that would benefit from embolization before surgery. Corpectomy and anterior

fusion is incorrect because immediate surgery is not needed here (intraosseous lesion causing

pain with a normal neurologic examination and mildly decreased range of motion). Radiation

and chemotherapy should not be administered before a diagnosis is ascertained.

RECOMMENDED READINGS

Rougraff BT, Kneisl JS, Simon MA. Skeletal metastases of unknown origin. A prospective study of a diagnostic strategy. J Bone Joint Surg

Am. 1993 Sep;75(9):1276-81. PubMed PMID: 8408149.

View Abstract at PubMed

Biermann JS, Holt GE, Lewis VO, Schwartz HS, Yaszemski MJ. Metastatic bone disease: diagnosis, evaluation, and treatment. J Bone

View Abstract at PubMed

Page 38: 2014 Musculoskeletal Tumors and Diseases

Figure 55a

Figure 55b

Figure 55c

Figure 55d

Figure 55e

Question 55 of 143

A 27-year-old incarcerated man was found to have a fungating mass on his anterolateral right

proximal thigh. A clinical photograph is shown in Figure 55a. T1- and T2-weighted coronal

MRI scans are shown in Figures 55b and 55c. The hematoxylin and eosin and CD34 stained

histology are shown in Figures 55d and 55e. What is the most likely diagnosis?

1- Squamous cell carcinoma

2- Melanoma

3- Dermatofibrosarcoma protuberans (DFSP)

4- Desmoid tumor

PREFERRED RESPONSE: 3- Dermatofibrosarcoma protuberans (DFSP)

DISCUSSION

The clinical photograph is characteristic for a large DFSP with lobules of tumor that often

fungate over a long period of time. The tumor has an infiltrative growth pattern and has a

high local recurrence rate if not widely resected. The tumor is characterized by the t(17;22)

translocation with the gene PDGFB to the collagen 1 alpha gene. The gene fusion product is

instrumental in the pathogenesis of the tumor. Histologically, the tumor is composed of

uniform fibroblasts arranged in a storiform pattern. CD34, the human progenitor cell antigen,

is positive. Although squamous cell carcinoma and melanoma can often fungate, the

protruding nodules of tumor clinically are not characteristic of those neoplasms. Aggressive

fibromatosis is much less cellular than DFSP and has less defined margins on MRI scan.

RECOMMENDED READINGS

Cai H, Wang Y, Wu J, Shi Y. Dermatofibrosarcoma protuberans: clinical diagnoses and treatment results of 260 cases in China. J Surg

Oncol. 2012 Feb;105(2):142-8. doi: 10.1002/jso.22000. Epub 2011 Aug 3. PubMed PMID: 21815146.

View Abstract at PubMed

McArthur G. Dermatofibrosarcoma protuberans: recent clinical progress. Ann Surg Oncol. 2007 Oct;14(10):2876-86. Epub 2007 Jul 24.

Review. PubMed PMID: 17647063. View Abstract at PubMed

Weiss SW, Goldblum JR. Fibrohistiocytic tumors of intermediate malignancy. In: Weiss SW, Goldblum JR, eds. Soft Tissue Tumors. 5th ed. Philadelphia, PA: Mosby Elsevier; 2008:371-382.

Page 39: 2014 Musculoskeletal Tumors and Diseases

Figure 56a

Figure 56b

Figure 56c

Question 56 of 143

Figures 56a and 56b are the axial short tau inversion recovery and T1 with contrast images of

a 7-month-old infant who is found to have a right scapular soft-tissue mass. On examination,

the mass is hard. A biopsy was performed and is shown in Figure 56c (hematoxylin and

eosin, 400x). What is the optimal treatment for this patient?

1- Intralesional excision

2- Marginal excision

3- Wide excision

4- Observation

PREFERRED RESPONSE: 3- Wide excision

DISCUSSION

The lesion seen here is infantile fibromatosis, and advanced imaging (ultrasonography, CT

scan, and MRI scan) reveals lesion extent. Involvement of adjacent structures is common,

reflecting the infiltrative growth pattern often seen in these lesions. MRI scan may show

characteristic features of prominent low- to intermediate-signal intensity and bands of low-

signal intensity representing highly collagenized tissue. However, fibromatoses with less

collagen and more cellularity may have nonspecific high-signal intensity on T2-weighted

images. Local recurrence is frequent after surgical resection because of the aggressive lesion

growth. This tumor is treated with wide excision, and chemotherapy is frequently employed

as an adjunct.

RECOMMENDED READINGS

Robbin MR, Murphey MD, Temple HT, Kransdorf MJ, Choi JJ. Imaging of musculoskeletal fibromatosis. Radiographics. 2001 May-

Jun;21(3):585-600. Review. PubMed PMID: 11353108. View Abstract at PubMed

Santoro A, Pannone G, Errico ME, Bifano D, Lastilla G, Bufo P, Loreto C, Donofrio V. Role of ß-catenin expression in paediatric mesenchymal lesions: a tissue microarray-based immunohistochemical study. Eur J Histochem. 2012 Jul 2;56(3):e25. doi:

10.4081/ejh.2012.e25. PubMed PMID: 23027341.

View Abstract at PubMed

Page 40: 2014 Musculoskeletal Tumors and Diseases

Figure 57a

Figure 57b

Figure 57c

Figure 57d

Figure 57e

Question 57 of 143

Figures 57a through 57d show the radiographs and T1- and T2-weighted MRI scan sequences

of the proximal femur of a 60-year-old man with progressive thigh pain. A review of

radiographs taken 3 years ago reveals that the lesion is new. The biopsy specimen is shown in

Figure 57e; staging studies show no other lesions, and local imaging confirms the process is

confined to bone (no soft-tissue mass). The next treatment step should include

1- hip disarticulation.

2- radiotherapy.

3- ifosfamide-based chemotherapy.

4- wide resection and reconstruction.

PREFERRED RESPONSE: 4- wide resection and reconstruction.

DISCUSSION

The radiographs and MRI scans show a cartilaginous lesion of the proximal femur with

cortical thickening and cortical invasion seen on axial MRI scan. The fact that the lesion is

new proves it is not an enchondroma (which would not be expected to arise de novo in an

adult and would not have such significant cortical changes). Biopsy reveals an intermediate-

grade chondrosarcoma with nuclear atypia in addition to permeation of trabecular bone. The

treatment of choice is surgical resection and reconstruction. Hip disarticulation is not

indicated because the lesion is confined to the bone without involvement of vessels or nerves.

Radiotherapy would be indicated for hematopoietic or radiosensitive malignancies, but

chondrosarcoma does not respond to radiation or chemotherapy. Curettage and bone cement

have been advocated as treatment for low-grade chondrosarcomas. However, the clear

nuclear atypia on the biopsy and the demonstration of intracortical involvement on the axial

MRI scan make this a less optimal treatment choice for this patient.

RECOMMENDED READINGS

Lee FY, Mankin HJ, Fondren G, Gebhardt MC, Springfield DS, Rosenberg AE, Jennings LC. Chondrosarcoma of bone: an assessment of

outcome. J Bone Joint Surg Am. 1999 Mar;81(3):326-38. PubMed PMID: 10199270.

View Abstract at PubMed

Leerapun T, Hugate RR, Inwards CY, Scully SP, Sim FH. Surgical management of conventional grade I chondrosarcoma of long bones.

Clin Orthop Relat Res. 2007 Oct;463:166-72. PubMed PMID: 17632422. View Abstract at PubMed

Page 41: 2014 Musculoskeletal Tumors and Diseases

Donati D, Colangeli S, Colangeli M, Di Bella C, Bertoni F. Surgical treatment of grade I central chondrosarcoma. Clin Orthop Relat Res.

2010 Feb;468(2):581-9. doi: 10.1007/s11999-009-1056-7. Epub 2009 Aug 29. PubMed PMID: 19727994. View Abstract at PubMed

Question 58 of 143

The characteristic translocation and genes involved in extraskeletal myxoid chondrosarcoma

is

1- t(11;12) EWS;FLI1.

2- t(12;16) TLS;CHOP.

3- t(9;22) EWS;CHN.

4- t(9;22) BCR-ABL.

PREFERRED RESPONSE: 3- t(9;22) EWS;CHN.

DISCUSSION

Extraskeletal myxoid chondrosarcoma is characterized by an in-frame fusion of the genes

EWS and TEC (also called CHN) involving chromosomes 9 and 22, respectively. Chronic

myelogenous leukemia is also characterized by a translocation involving chromosomes 9 and

22; however, the fused genes are different. t(11;12) EWS/FLI1 is characteristic of Ewing

sarcoma/t(12;16) TLS; CHOP is characteristic of myxoid liposarcoma.

RECOMMENDED READINGS

Hinrichs SH, Jaramillo MA, Gumerlock PH, Gardner MB, Lewis JP, Freeman AE. Myxoid chondrosarcoma with a translocation involving

chromosomes 9 and 22. Cancer Genet Cytogenet. 1985 Jan 15;14(3-4):219-26. PubMed PMID: 3967207. View Abstract at PubMed

Labelle Y, Bussières J, Courjal F, Goldring MB. The EWS/TEC fusion protein encoded by the t(9;22) chromosomal translocation in human chondrosarcomas is a highly potent transcriptional activator. Oncogene. 1999 May 27;18(21):3303-8. PubMed PMID: 10359536.

View Abstract at PubMed

Figure 59a

Figure 59b

Figure 59c

Question 59 of 143

Figures 59a and 59b are the axial T2 and T1 with contrast MRI scans of a 32-year-old woman

who has a 10-year history of pain and a 1-year history of progressive swelling in her right leg.

The histopathology is shown in Figure 59c. A radiograph of her leg showed no

mineralizations or osseous erosions. The chromosomal abnormality that is associated with

this disease is

Page 42: 2014 Musculoskeletal Tumors and Diseases

1- t(11;22).

2- t(2;13).

3- t(X;18).

4- t(12;16).

PREFERRED RESPONSE: 3- t(X;18).

DISCUSSION

The imaging shows a soft-tissue mass with histology showing a synovial sarcoma. This

histology is associated with t(X;18). Ewing sarcoma is associated with t(11;22); alveolar

rhabdomyosarcoma t(2;13); and myxoid liposarcoma t(12;16). Although the presented

monophasic histology could be consistent with other sarcomas, it does not have the

characteristics of Ewing sarcoma (small round blue cells), alveolar rhabdomysarcoma

(alveolar pattern), or myxoid liposarcoma (myxoid stroma and signet ring lipoblasts with a

plexiform capillary network).

RECOMMENDED READINGS

Jones BC, Sundaram M, Kransdorf MJ. Synovial sarcoma: MR imaging findings in 34 patients. AJR Am J Roentgenol. 1993

Oct;161(4):827-30. PubMed PMID: 8396848. View Abstract at PubMed

Sandberg AA. Cytogenetics and molecular genetics of bone and soft-tissue tumors. Am J Med Genet. 2002 Oct 30;115(3):189-93. Review.

PubMed PMID: 12407700.

View Abstract at PubMed

CLINICAL SITUATION FOR QUESTIONS 60 THROUGH 63

A 45-year-old woman has an enlarging buttock mass. The mass is 12 cm and nonpainful. The

patient first noticed it about 6 months after she had a low-impact fall. The general surgeon

evaluating the patient felt this mass could be either a lipoma or a hematoma. The patient

underwent a surgical procedure to remove the mass.

Question 60 of 143

After evaluating the mass with a history and examination, what is the most appropriate next

treatment step?

1- Observation

2- Imaging studies (MRI scan or CT scan)

3- Surgical excision

4- Open biopsy

PREFERRED RESPONSE: 2- Imaging studies (MRI scan or CT scan)

Question 61 of 143

Page 43: 2014 Musculoskeletal Tumors and Diseases

A surgical excision performed on this patient should be done with which principle in mind to

minimize contamination of soft tissues?

1- Meticulous hemostasis and closure

2- Traversing multiple muscle compartments

3- Transverse incision

4- Exposing neurovascular structures

PREFERRED RESPONSE: 1- Meticulous hemostasis and closure

Question 62 of 143

Which description best characterizes an unplanned excision of a soft-tissue sarcoma?

1- The length of the incision is excessive.

2- The incision is oriented transversely on the extremity.

3- No imaging was obtained before surgery.

4- No radiation was given before surgery.

PREFERRED RESPONSE: 3- No imaging was obtained before surgery.

Question 63 of 143

What is the most common detrimental impact of an unplanned excision of a high-grade soft-

tissue sarcoma?

1- Decreased mortality

2- Decreased recurrence

3- Increased wound complications

4- Increased functional outcome

PREFERRED RESPONSE: 3- Increased wound complications

DISCUSSION

This is an example of an unplanned excision. The patient has a history of trauma and a

painless soft-tissue mass. The most common soft-tissue masses are benign; consequently,

nononcologic surgeons may not expect a malignant sarcoma. The patient was taken to

surgery with appropriate imaging and without a biopsy. If the imaging is conclusive a biopsy

is not always indicated, particularly for low-grade lipomatous tumors. The surgical resection

was performed through a transverse incision. It is recommended that the incision be made

longitudinally in the event a wider resection is needed. The grade of the tumor is the most

important aspect. Benign and low-grade tumors may be observed, whereas intermediate and

high-grade tumors require a wide resection and likely perioperative radiation. The appropriate

work-up for a soft-tissue mass is a careful history and examination accompanied by

radiographs and cross-sectional imaging. If a biopsy is needed, it should be performed or

Page 44: 2014 Musculoskeletal Tumors and Diseases

supervised by the surgeon, who would do the final resection if it proves to be a sarcoma. The

most common impact of an unplanned excision is wound complications. This is because of

the need for a wider surgical resection and adjuvant radiation. Mortality and recurrence have

not been shown to be increased with statistical significance. Functional outcome may be

compromised; however, no studies confirm that functional outcome is poorer in patients who

undergo unplanned excision.

RECOMMENDED READINGS

Qureshi YA, Huddy JR, Miller JD, Strauss DC, Thomas JM, Hayes AJ. Unplanned excision of soft tissue sarcoma results in increased rates

of local recurrence despite full further oncological treatment. Ann Surg Oncol. 2012 Mar;19(3):871-7. doi: 10.1245/s10434-011-1876-z. Epub 2011 Jul 27. PubMed PMID: 21792512.

View Abstract at PubMed

Arai E, Nishida Y, Tsukushi S, Wasa J, Ishiguro N. Clinical and treatment outcomes of planned and unplanned excisions of soft tissue

sarcomas. Clin Orthop Relat Res. 2010 Nov;468(11):3028-34. doi: 10.1007/s11999-010-1392-7. PubMed PMID: 20496020.

View Abstract at PubMed

Siegel HJ, Brown O, Lopez-Ben R, Siegal GP. Unplanned surgical excision of extremity soft tissue sarcomas: patient profile and referral

patterns. J Surg Orthop Adv. 2009 Summer;18(2):93-8. PubMed PMID: 19602337. View Abstract at PubMed

Figure 64a

Figure 64b

Figure 64c

CLINICAL SITUATION FOR QUESTIONS 64 THROUGH 66

Figures 64a through 64c are the radiograph, MRI scan, and histology of a 53-year-old man

with medial knee pain and swelling below the knee.

Question 64 of 143

What is the origin of the lesion seen on imaging?

1- A meniscal tear

2- A parameniscal cyst

Page 45: 2014 Musculoskeletal Tumors and Diseases

3- Elastin deficiency

4- Translocation x;18

PREFERRED RESPONSE: 4- Translocation x;18

Question 65 of 143

Arthroscopy is performed, the palpable lesion is aspirated, and no fluid is obtained. What is

the best next treatment step?

1- Marginal resection of the cyst through a transverse incision

2- Marginal resection of the cyst through a longitudinal incision

3- Arthroscopic evaluation of the cyst

4- Biopsy

PREFERRED RESPONSE: 4- Biopsy

Question 66 of 143

Histology of the lesion is shown in Figure 64c. The best next treatment step is

1- radiation.

2- radiation and surgery.

3- chemotherapy.

4- observation.

PREFERRED RESPONSE: 2- radiation and surgery.

DISCUSSION

Parameniscal cysts and soft-tissue sarcomas can easily be confused, especially when there is

an intra-articular abnormality. This patient has a synovial sarcoma, which has an x;18

translocation abnormality. The lack of communication with the joint; no meniscal tear on

MRI scan; a lesion that has signal characteristics that are not the same as muscle, fat, or fluid;

and aspiration that yields no fluid are all characteristics of a soft-tissue sarcoma. A

parameniscal cyst has a signal characteristic that is identical to the joint fluid on MRI scan

and has a visible connection to the joint. When a lesion is encountered that may have

malignant potential, a biopsy is imperative. Principles of an extremity biopsy—longitudinal

incision, strict hemostasis, minimizing tissue plane dissection, and ensuring appropriate

tissue—must be observed. Marginal resection of a soft-tissue sarcoma, especially through a

transverse incision, would not be appropriate care. A soft-tissue sarcoma is treated with

radiation and surgery. Chemotherapy is controversial in this setting with no long-term

benefit.

RECOMMENDED READINGS

Page 46: 2014 Musculoskeletal Tumors and Diseases

Muscolo DL, Ayerza MA, Makino A, Costa-Paz M, Aponte-Tinao LA. Tumors about the knee misdiagnosed as athletic injuries. J Bone

Joint Surg Am. 2003 Jul;85-A(7):1209-14. PubMed PMID: 12851344. View Abstract at PubMed

Damron TA, Morris C, Rougraff B, Tamurian R. Diagnosis and treatment of joint-related tumors that mimic sports-related injuries. Instr Course Lect. 2009;58:833-47. PubMed PMID: 19385590.

View Abstract at PubMed

Gilbert NF, Cannon CP, Lin PP, Lewis VO. Soft-tissue sarcoma. J Am Acad Orthop Surg. 2009 Jan;17(1):40-7. Review. PubMed PMID:

19136426.

View Abstract at PubMed

Figure 67a

Figure 67b

Figure 67c

Question 67 of 143

A 26-year-old woman has had a slow-growing and painful mass at the base of her ring finger

for several months. Radiographs of the affected digit show no mineralization or erosions of

the underlying bone. An axial T1 MRI scan is shown in Figure 67a, and a corresponding

short tau inversion recovery image is shown in Figure 67b. A coronal T1 MRI scan with

contrast is shown in Figure 67c. The best next treatment step is

1- referral to a sarcoma center.

2- observation.

3- excisional biopsy.

4- marginal excision.

PREFERRED RESPONSE: 1- referral to a sarcoma center.

DISCUSSION

The lesion seen in the images is larger than 1.5 cm and growing. Therefore, a malignant

growth must be considered. Although malignant tumors of the hand are rare and clear-cell

sarcoma is rarer still, the most common clinical symptom is a small painless mass. Curative

excision or amputation is the treatment of choice. Observation of a malignant lesion is not

acceptable treatment, and neither an excisional biopsy nor marginal excision can achieve the

wide excision necessary for this patient.

RECOMMENDED READINGS

Brien EW, Terek RM, Geer RJ, Caldwell G, Brennan MF, Healey JH. Treatment of soft-tissue sarcomas of the hand. J Bone Joint Surg Am. 1995 Apr;77(4):564-71. PubMed PMID: 7713973.

View Abstract at PubMed

Page 47: 2014 Musculoskeletal Tumors and Diseases

Plate AM, Steiner G, Posner MA. Malignant tumors of the hand and wrist. J Am Acad Orthop Surg. 2006 Nov;14(12):680-92. PubMed

PMID: 17077340. View Abstract at PubMed

Question 68 of 143

What is the most specific immunohistochemistry staining pattern that confirms the diagnosis

of desmoid tumor?

1- Membranous beta-catenin staining

2- Nuclear beta-catenin staining

3- Nuclear SMAD4 staining

4- Vimentin positivity

PREFERRED RESPONSE: 2- Nuclear beta-catenin staining

DISCUSSION

Desmoid tumors are characterized by aberrant activation of Wnt signaling resulting from

mutations in the adenomatous polyposis coli and beta-catenin genes. Although a nuclear beta-

catenin staining pattern can be observed in other tumor types, it is highly specific for desmoid

tumor. Vimentin positivity indicates that a lesion is a mesenchymal neoplasm, but it is not

specific for desmoid tumor.

RECOMMENDED READINGS

Ng TL, Gown AM, Barry TS, Cheang MC, Chan AK, Turbin DA, Hsu FD, West RB, Nielsen TO. Nuclear beta-catenin in mesenchymal

tumors. Mod Pathol. 2005 Jan;18(1):68-74. PubMed PMID: 15375433. View Abstract at PubMed

Tejpar S, Nollet F, Li C, Wunder JS, Michils G, dal Cin P, Van Cutsem E, Bapat B, van Roy F, Cassiman JJ, Alman BA. Predominance of beta-catenin mutations and beta-catenin dysregulation in sporadic aggressive fibromatosis (desmoid tumor). Oncogene. 1999 Nov

11;18(47):6615-20. PubMed PMID: 10597266.

View Abstract at PubMed

Figure 69a

Figure 69b

Figure 69c

Figure 69d

Question 69 of 143

Page 48: 2014 Musculoskeletal Tumors and Diseases

A 30-year-old woman has progressive gait instability, back pain, and urinary retention.

Figures 69a and 69b show the axial T2 and postcontrast MRI scans taken at the level of T11,

and Figure 69c shows the sagittal T1-weighted image. Representative histology is shown in

Figure 69d. The patient remains ambulatory, but symptoms have progressed during the last

week and she is beginning to feel weakness in her legs. Examination is notable for decreased

rectal tone, lower-extremity hyperreflexia and clonus, and 4/5 motor strength throughout the

lower extremities. What is the most appropriate treatment recommendation for this patient?

1- Margin-free en bloc spondylectomy of T11

2- Radiation therapy

3- Radiation therapy followed by anterior corpectomy and fusion

4- Transpedicular decompression and posterior stabilization followed by radiation therapy

PREFERRED RESPONSE: 4- Transpedicular decompression and posterior stabilization

followed by radiation therapy

DISCUSSION

This patient has symptomatic metastatic epidural spinal cord compression from synovial

sarcoma with pathologic fracture and epidural extension of disease. Patchell and associates

demonstrated outcomes with direct decompressive surgery followed by radiation therapy that

were superior to radiation alone for patients presenting in this manner; note that the trial

excluded patients with exquisitely radiosensitive malignancies such as lymphoma and germ-

cell tumors. Ibrahim and associates confirmed these results in a prospective multicenter

observational study. The sagittal T1-weighted image shows other lesions at T1 and T5,

confirming a metastatic process. The histology shows synovial sarcoma and excludes

hematopoietic or germ-cell malignancy (very radiosensitive). Margin-free en bloc

spondylectomy would be difficult to achieve considering extension of tumor into the

posterior elements, degree of epidural compression, and abutment of the aorta. More

important, it is difficult to justify the magnitude and risk associated with this procedure in a

patient with multiple metastatic lesions. Radiation therapy alone and radiation followed by

surgery were shown in Patchell’s study to have inferior outcomes. A patient with a

progressive neurologic deficit should have surgery before radiotherapy.

RECOMMENDED READINGS

Patchell RA, Tibbs PA, Regine WF, Payne R, Saris S, Kryscio RJ, Mohiuddin M, Young B. Direct decompressive surgical resection in the

treatment of spinal cord compression caused by metastatic cancer: a randomised trial. Lancet. 2005 Aug 20-26;366(9486):643-8. PubMed

PMID: 16112300. View Abstract at PubMed

Ibrahim A, Crockard A, Antonietti P, Boriani S, Bünger C, Gasbarrini A, Grejs A, Harms J, Kawahara N, Mazel C, Melcher R, Tomita K. Does spinal surgery improve the quality of life for those with extradural (spinal) osseous metastases? An international multicenter

prospective observational study of 223 patients. Invited submission from the Joint Section Meeting on Disorders of the Spine and Peripheral

Nerves, March 2007. J Neurosurg Spine. 2008 Mar;8(3):271-8. doi: 10.3171/SPI/2008/8/3/271. PubMed PMID: 18312079. View Abstract at PubMed

Wang JC, Boland P, Mitra N, Yamada Y, Lis E, Stubblefield M, Bilsky MH. Single-stage posterolateral transpedicular approach for resection of epidural metastatic spine tumors involving the vertebral body with circumferential reconstruction: results in 140 patients.

Invited submission from the Joint Section Meeting on Disorders of the Spine and Peripheral Nerves, March 2004. J Neurosurg Spine. 2004

Oct;1(3):287-98. PubMed PMID: 15478367. View Abstract at PubMed

Page 49: 2014 Musculoskeletal Tumors and Diseases

Question 70 of 143

Giant-cell tumor of bone usually involves the epiphysis of long bones. What is the next most

common type of tumor involving this anatomical location?

1- Conventional chondrosarcoma

2- Aneurysmal bone cyst

3- Chondroblastoma

4- Osteoblastoma

PREFERRED RESPONSE: 3- Chondroblastoma

DISCUSSION

Giant-cell tumor is the most common lesion involving the epiphysis, although its epicenter is

usually in the metaphysis, and in the rare case of giant-cell tumor occurring in a skeletally

immature patient, giant-cell tumor is located in the metaphysis. It also can involve the flat

bone of the pelvis and sacrum. Chondroblastoma exclusively occurs in the epiphysis of

skeletally immature patients. It is more common than the other responses. Although

conventional chondrosarcoma does not involve the epiphysis, clear-cell chondrosarcoma

involves the epiphysis as well; however, it is rare and less common than chondroblastoma.

Aneurysmal bone may occur in the epiphysis; however, it is often seen with other benign

tumors such as chondroblastoma or giant-cell tumor. It is less common in this location than

chondroblastoma. Osteoblastoma does not classically appear at the epiphysis. It is more

commonly seen in the spine or the diaphysis of long bones.

RECOMMENDED READINGS

Sailhan F, Chotel F, Parot R; SOFOP. Chondroblastoma of bone in a pediatric population. J Bone Joint Surg Am. 2009 Sep;91(9):2159-68.

doi: 10.2106/JBJS.H.00657. PubMed PMID: 19723993. View Abstract at PubMed

Ramappa AJ, Lee FY, Tang P, Carlson JR, Gebhardt MC, Mankin HJ. Chondroblastoma of bone. J Bone Joint Surg Am. 2000 Aug;82-A(8):1140-5. PubMed PMID: 10954104.

View Abstract at PubMed

Figure 71a

Figure 71b

Figure 71d

Figure 71e

Page 50: 2014 Musculoskeletal Tumors and Diseases

Figure 71c

CLINICAL SITUATION FOR QUESTIONS 71 THROUGH 73

Figures 71a through 71e are the radiographs, MRI scan, and CT scans of a 14-year-old-boy

who has cyclical pain in his thigh. His symptoms began approximately 6 months ago. He

complains of increased pain when he runs and also of pain that wakes him at night. This pain

is relieved by nonsteroidal anti-inflammatory drugs (NSAIDs).

Question 71 of 143

What is the most likely diagnosis?

1- Stress fracture

2- Osteomyelitis

3- Osteoid osteoma

4- Eosinophilic granuloma

PREFERRED RESPONSE: 3- Osteoid osteoma

Question 72 of 143

Contemporary definitive treatment for this patient is

1- NSAIDs.

2- radiofrequency ablation.

3- intravenous antibiotics.

4- burr-down technique resection.

PREFERRED RESPONSE: 2- radiofrequency ablation.

Question 73 of 143

What is the etiology of the pain associated with this lesion?

1- Prostaglandin production

2- Gram-positive cocci

3- Osteoclast activation

4- Loss of structural integrity of the bone

PREFERRED RESPONSE: 1- Prostaglandin production

DISCUSSION

Page 51: 2014 Musculoskeletal Tumors and Diseases

Osteoid osteomas are most common in adolescent patients and in boys. Although these

tumors can be seen in any bone, almost half are located in the femur and tibia. The significant

inflammatory response to this tumor is presumed to be secondary to high levels of

prostaglandin production. Characteristic pain at night is relieved by aspirin or NSAIDs. Pain

can also be associated with activities and is characterized as being boring and sharp.

Radiographs often show distinctive features of thickened bone surrounding a small central

core of lower density (a nidus). Thin-cut CT scan is the imaging modality of choice for

visualizing the nidus, which typically lies within the cortex of the bone. An MRI scan may be

misleading because of the extensive edema associated with this lesion, bringing other

differential diagnoses into question. A bone scan is also associated with significant uptake but

is nonspecific. Several treatment options are available, including observation and

management with NSAIDs for pain control, with the premise that the lesion will “burn out.”

This may take several years to occur and few patients are content with this treatment regimen.

Surgical resection can be accomplished with en bloc resection, but this approach is fraught

with complications associated with resecting large segments of bone. A fluoroscopically

assisted “burr-down” technique is effective in removing the nidus and minimizing the stress

riser effect to the bone but requires an open surgery. Radiofrequency ablation is the current

first-line treatment modality for osteoid osteoma because of its high efficacy and low

morbidity.

CT-guided radiofrequency ablation removal of the nidus has been successfully used with few

recurrences or complications. This is a lesser procedure, compared to en bloc resection or

intralesional surgery, and may be performed under general anesthesia or a regional block with

sedation. The site of the tumor is identified using fine CT sections, and a radiofrequency

probe is inserted into the tumor. The probe heats the tumor tissues to approximately 90°C,

effectively obliterating the nidus with minimal damage to surrounding tissues. The nidus is

adequately removed in 96% of patients after 1 treatment. The procedure is routinely

performed on an outpatient basis by either an interventional radiologist or an orthopaedic

surgeon. Most procedures take approximately 2 hours, followed by a 2-hour recovery period,

after which the patient may go home with a mild pain reliever. In most cases, the patient may

return to work or school in a few days with some restrictions.

RECOMMENDED READINGS

Rimondi E, Bianchi G, Malaguti MC, Ciminari R, Del Baldo A, Mercuri M, Albisinni U. Radiofrequency thermoablation of primary non-

spinal osteoid osteoma: optimization of the procedure. Eur Radiol. 2005 Jul;15(7):1393-9. Epub 2005 Mar 9. PubMed PMID: 15756555. View Abstract at PubMed

Ward WG, Eckardt JJ, Shayestehfar S, Mirra J, Grogan T, Oppenheim W. Osteoid osteoma diagnosis and management with low morbidity. Clin Orthop Relat Res. 1993 Jun;(291):229-35. PubMed PMID: 8504605.

View Abstract at PubMed

Ilyas I, Younge DA. Medical management of osteoid osteoma. Can J Surg. 2002 Dec;45(6):435-7. PubMed PMID: 12500919.

View Abstract at PubMed

Page 52: 2014 Musculoskeletal Tumors and Diseases

Figure 74

Question 74 of 143

Figure 74 is the radiograph of an 11-year-old boy with pain in his left arm. Prognosis is most

influenced by

1- stage at presentation.

2- grade at presentation.

3- response to neoadjuvant chemotherapy.

4- histologic subtype.

PREFERRED RESPONSE: 1- stage at presentation.

DISCUSSION

The imaging is characteristic for high-grade osteosarcoma. In patients with osteosarcoma,

stage at presentation has the most prognostic impact; clinically detectable mestastases at

presentation has a very poor prognosis. The other factors all influence prognosis in patients

without metastatic disease. Low-grade nonmetastatic tumors have the best prognosis and are

treated with surgery alone. In high-grade tumors, histologic response to neoadjuvant

chemotherapy is predictive of survival. Response to chemotherapy, distal appendicular sites,

and female gender were associated with improved survival in the study by Whelan and

associates of 1067 patients with localized extremity osteosarcoma. Increased tumor volume

after chemotherapy had an adverse effect on survival independent of histologic response to

chemotherapy. Age, high levels of alkaline phosphatase, tumor volume, 2-drug regimen,

surgical margins, and histologic response to chemotherapy were important prognostic factors.

RECOMMENDED READINGS

Whelan JS, Jinks RC, McTiernan A, Sydes MR, Hook JM, Trani L, Uscinska B, Bramwell V, Lewis IJ, Nooij MA, van Glabbeke M,

Grimer RJ, Hogendoorn PC, Taminiau AH, Gelderblom H. Survival from high-grade localised extremity osteosarcoma: combined results

and prognostic factors from three European Osteosarcoma Intergroup randomised controlled trials. Ann Oncol. 2012 Jun;23(6):1607-16. doi: 10.1093/annonc/mdr491. Epub 2011 Oct 19. PubMed PMID: 22015453.

View Abstract at PubMed

Kim MS, Lee SY, Cho WH, Song WS, Koh JS, Lee JA, Yoo JY, Jung ST, Jeon DG. Effect of increases in tumor volume after neoadjuvant

chemotherapy on the outcome of stage II osteosarcoma regardless of histological response. J Orthop Sci. 2009 May;14(3):292-7. doi:

10.1007/s00776-009-1334-y. Epub 2009 Jun 5. PubMed PMID: 19499296. View Abstract at PubMed

Bacci G, Longhi A, Versari M, Mercuri M, Briccoli A, Picci P. Prognostic factors for osteosarcoma of the extremity treated with neoadjuvant chemotherapy: 15-year experience in 789 patients treated at a single institution. Cancer. 2006 Mar 1;106(5):1154-61. PubMed

PMID: 16421923.

View Abstract at PubMed

Page 53: 2014 Musculoskeletal Tumors and Diseases

Figure 75a

Figure 75b

Figure 75c

Figure 75d

Question 75 of 143

Figures 75a through 75d are the radiograph, CT scan, bone scan, and biopsy of a 45-year-old

man who has had a several-month history of progressive pain in his right hip and groin

region. Based on these images and histology, what is the most appropriate treatment?

1- Wide resection

2- Curetting and bone grafting

3- Percutaneous cementation and radiotherapy

4- Chemotherapy and radiotherapy

PREFERRED RESPONSE: 1- Wide resection

DISCUSSION

The imaging (which includes a clear chest CT scan) demonstrates a lytic periacetabular lesion

with internal matrix production and a cellular cartilage lesion on histology, diagnostic for a

chondrosarcoma with negative staging. Wide excision with limb salvage is appropriate for a

lesion without evidence of neurovascular encasement. Although used for select low-grade

cartilage lesions in the extremity, curettage is not indicated for pelvic chondrosarcomas.

Radiation and chemotherapy are not used in the treatment of conventional chondrosarcoma.

RECOMMENDED READINGS

Han I, Lee YM, Cho HS, Oh JH, Lee SH, Kim HS. Outcome after surgical treatment of pelvic sarcomas. Clin Orthop Surg. 2010 Sep;2(3):160-6. doi: 10.4055/cios.2010.2.3.160. Epub 2010 Aug 3. PubMed PMID: 20808587.

View Abstract at PubMed

Deloin X, Dumaine V, Biau D, Karoubi M, Babinet A, Tomeno B, Anract P. Pelvic chondrosarcomas: surgical treatment options. Orthop

Traumatol Surg Res. 2009 Oct;95(6):393-401. doi: 10.1016/j.otsr.2009.05.004. Epub 2009 Oct 3. PubMed PMID: 19801212.

View Abstract at PubMed

Page 54: 2014 Musculoskeletal Tumors and Diseases

Figure 76

Question 76 of 143

Figure 76 is the radiograph of a 77-year-old patient with a history of myeloma who has had

severe arm pain after opening a jar. Pain was present for 3 months prior to injury. The most

biomechanically stable construct for this fracture is

1- intramedullary nailing (IMN).

2- IMN and cement.

3- plate.

4- plate and cement.

PREFERRED RESPONSE: 4- plate and cement.

DISCUSSION

Although intramedullary nails or plates with screws, either of which may be combined with

cement, may be used as internal fixation for a displaced humeral diaphyseal pathologic

fracture, torsional performance is best biomechanically when the canal is filled with cement

and fixed with a plate. In the largest comparative biomechanical study to date, 40 artificial

humeri were divided into 5 different constructs and tested in torsion. The construct that

resisted the largest load to failure was the construct at which the canal and tumor defect were

filled with bone cement and the screws were inserted into dry cement. That being said, the

weaknesses of the plate and cement technique include the potential for poor quality bone

adjacent to and extending away from the fracture site and difficulty in protecting the entire

bone from progression of local disease. Numerous factors beyond just the biomechanics,

including the patient’s disease load and comorbidities, the underlying primary disease and

responsiveness to radiotherapy and other adjuvant treatments, and the location and local

extent of disease, must be considered when determining the best operative technique to

employ in each patient with a pathologic fracture.

RECOMMENDED READINGS

Al-Jahwari A, Schemitsch EH, Wunder JS, Ferguson PC, Zdero R. The biomechanical effect of torsion on humeral shaft repair techniques

for completed pathological fractures. J Biomech Eng. 2012 Feb;134(2):024501. doi: 10.1115/1.4005696. PubMed PMID: 22482676. View Abstract at PubMed

Weiss KR, Bhumbra R, Biau DJ, Griffin AM, Deheshi B, Wunder JS, Ferguson PC. Fixation of pathological humeral fractures by the cemented plate technique. J Bone Joint Surg Br. 2011 Aug;93(8):1093-7. doi: 10.1302/0301-620X.93B8.26194. PubMed PMID: 21768635.

View Abstract at PubMed

Page 55: 2014 Musculoskeletal Tumors and Diseases

Figure 77a

Figure 77b

Figure 77c

CLINICAL SITUATION FOR QUESTIONS 77 THROUGH 79

Figures 77a through 77c are the radiograph and MRI scans of a 45-year-old woman who

enjoys dancing and has had left ankle pain for approximately 1 month with minimal trauma.

She has slightly limited ankle dorsiflexion with a mechanical block with pain.

Question 77 of 143

What process most likely led to the pathology?

1- Synovial metaplasia

2- Posttraumatic heterotopic ossification

3- Malignant chondroid development

4- Neoplastic proliferation of a mixed epithelial and spindle-cell lesion

PREFERRED RESPONSE: 1- Synovial metaplasia

Question 78 of 143

An anterior approach to the ankle is chosen to resect this lesion. From medial to lateral, what

are the structures just below the level of the ankle joint?

1- Extensor hallucis longus (EHL) tendon, tibialis anterior (TA) tendon, deep peroneal nerve,

anterior tibial artery, extensor digitorum longus (EDL) tendon

2- TA, EHL, anterior tibial artery, deep peroneal nerve, EDL

3- TA, EHL, EDL, deep peroneal nerve, anterior tibial artery

4- TA, deep peroneal nerve, anterior tibial artery, EHL, EDL

PREFERRED RESPONSE: 2- TA, EHL, anterior tibial artery, deep peroneal nerve, EDL

Question 79 of 143

Page 56: 2014 Musculoskeletal Tumors and Diseases

The patient undergoes recommended surgery and minimal joint damage is found. How

should she be counseled regarding her future prognosis?

1- High risk for local recurrence, moderate risk for metastases

2- High risk for local recurrence, no risk for metastases

3- Low risk for local recurrence, moderate risk for metastases

4- Low risk for local recurrence, no risk for metastases

PREFERRED RESPONSE: 4- Low risk for local recurrence, no risk for metastases

DISCUSSION

This patient has history and imaging consistent with synovial chondromatosis (synovial

osteochondromatosis). This condition most often is seen in larger joints, particularly the knee,

but has been well described in the ankle. The most common cause is primary synovial

chondromatosis, which is a benign proliferative chondroid metaplasia of the synovium. These

can exist in various states of calcification. In this patient, the radiographs and MRI scan

reveal multiple well-calcified lesions in the anterior portion of the joint with an effusion.

Secondary synovial chondromatosis can occur after trauma in the joint but is most likely

related to degenerative changes in the articular cartilage or meniscus/labral tissue.

Heterotopic ossification is rarely intra-articular, and would more likely have only 1 mass.

Neoplastic chondroid development would be a chondrosarcoma, which is exceedingly rare in

an intra-articular location, and, even with calcification, would be a solitary mass. Synovial

sarcoma is often seen in the foot and ankle, but is usually extra-articular nonsynovial with a

mixed epithelial and spindle-cell morphology.

Treatment of synovial chondromatosis is resection of the lesions and synovectomy.

Combined anterior and posterior approaches often can be used, and arthroscopy often is

helpful. Without significant removal of joint structures, there should be minimal instability

after surgery. A single surgery should handle the problem, but there is a small chance of local

recurrence. As a benign process, synovial chondromatosis does not metastasize.

RECOMMENDED READINGS

Holm CL. Primary synovial chondromatosis of the ankle. A case report. J Bone Joint Surg Am. 1976 Sep;58(6):878-80. PubMed PMID: 956234.

View Abstract at PubMed

Bojanic I, Bergovec M, Smoljanovic T. Combined anterior and posterior arthroscopic portals for loose body removal and synovectomy for

synovial chondromatosis. Foot Ankle Int. 2009 Nov;30(11):1120-3. doi: 10.3113/FAI.2009.1120. PubMed PMID: 19912726.

View Abstract at PubMed

Page 57: 2014 Musculoskeletal Tumors and Diseases

Figure 80a

Figure 80b

Figure 80c

Figure 80d

Question 80 of 143

Figures 80a through 80d are the radiographs and MRI scans of a 16-year-old girl who

requires crutches because she is unable to bear weight on her right lower extremity. The pain

has progressed over 2 months despite nonsurgical treatment. Treatment at this point should

include

1- continued observation.

2- aspiration and injection with methylprednisolone.

3- curettage and grafting.

4- wide margin resection.

PREFERRED RESPONSE: 3- curettage and grafting.

DISCUSSION

This lesion represents a nonossifying fibroma (NOF). In general, NOFs are found incidentally

and require no treatment other than serial follow up for a period of time to document stability.

However, when they are painful, particularly with weight bearing, the likelihood of

impending or evolving pathological fracture must be considered. In this patient, curettage and

grafting with or without internal fixation is the surgical treatment of choice. Although

aspiration and injection with methylprednisolone is appropriate for unicameral bone cysts, it

has little role in the management of NOF. Wide margin resection is reserved for malignancy,

which is not evident in this clinical scenario.

RECOMMENDED READINGS

Moretti VM, Slotcavage RL, Crawford EA, Lackman RD, Ogilvie CM. Curettage and graft alleviates athletic-limiting pain in benign lytic

bone lesions. Clin Orthop Relat Res. 2011 Jan;469(1):283-8. doi: 10.1007/s11999-010-1374-9. Epub 2010 May 12. PubMed PMID: 20461483.

View Abstract at PubMed

Vanel D, Ruggieri P, Ferrari S, Picci P, Gambarotti M, Staals E, Alberghini M. The incidental skeletal lesion: ignore or explore? Cancer

Imaging. 2009 Oct 2;9 Spec No A:S38-43. doi: 10.1102/1470-7330.2009.9009. Review. PubMed PMID: 19965292.

View Abstract at PubMed

Vlychou M, Athanasou NA. Radiological and pathological diagnosis of paediatric bone tumours and tumour-like lesions. Pathology. 2008

Feb;40(2):196-216. doi: 10.1080/00313020701813784. Review. PubMed PMID: 18203042. View Abstract at PubMed

Page 58: 2014 Musculoskeletal Tumors and Diseases

Question 81 of 143

A previously healthy 60-year-old woman has a T5 pathologic compression fracture secondary

to metastatic renal cell carcinoma. She has myelopathy with severe pain upon sitting from a

supine position. The adjacent thoracic levels are unaffected. On MRI scan, there is minimal

space available for the cord because of epidural involvement. What is the optimal treatment

method at this time?

1- Surgical decompression and instrumented fusion

2- Surgical decompression without fusion

3- Image-guided intensity-modulated radiation therapy

4- Chemotherapy

PREFERRED RESPONSE: 1- Surgical decompression and instrumented fusion

DISCUSSION

Data strongly support surgical treatment for spinal metastases when neurologic compromise

and/or mechanical instability are present. In this patient, pain upon sitting from a supine

position is a sign of mechanical instability. Although radiation therapy and chemotherapy are

appropriate for the treatment of spinal metastases in other circumstances, the optimal

treatment method in this case is to surgically decompress the affected level and restore

mechanical stability by fusing the motion segment.

RECOMMENDED READINGS

Patchell RA, Tibbs PA, Regine WF, Payne R, Saris S, Kryscio RJ, Mohiuddin M, Young B. Direct decompressive surgical resection in the

treatment of spinal cord compression caused by metastatic cancer: a randomised trial. Lancet. 2005 Aug 20-26;366(9486):643-8. PubMed PMID: 16112300.

View Abstract at PubMed

Furlan JC, Chan KK, Sandoval GA, Lam KC, Klinger CA, Patchell RA, Laporte A, Fehlings MG. The combined use of surgery and

radiotherapy to treat patients with epidural cord compression due to metastatic disease: a cost-utility analysis. Neuro Oncol. 2012

May;14(5):631-40. doi: 10.1093/neuonc/nos062. Epub 2012 Apr 14. PubMed PMID: 22505658. View Abstract at PubMed

Figure 82a

Figure 82b

Figure 82c

Figure 82d

Question 82 of 143

Page 59: 2014 Musculoskeletal Tumors and Diseases

Figures 82a and 82b are the MRI scans consisting of a T2 coronal sequence and axial T1

pulse sequence of a 38-year-old man who has had right thigh pain and a mass for 4 months

since he pulled his hamstring. The presumed diagnosis considering his clinical history and

evaluation of the MRI scan was hematoma and the mass was evacuated. The histology is

shown in Figures 82c and 82d. Next treatment steps should include

1- physical therapy to accelerate healing and improve function.

2- a complete hematologic work-up to evaluate a bleeding disorder.

3- tumor bed excision and radiation to reduce local recurrence.

4- radiotherapy to complete definitive treatment of this problem.

PREFERRED RESPONSE: 3- tumor bed excision and radiation to reduce local recurrence.

DISCUSSION

Unplanned excisions of soft-tissue sarcomas occur with alarming frequency and result in high

rates of residual disease, potentially affecting local control and local morbidity. Patients with

unplanned excisions who undergo limb-salvage procedures require more extensive surgical

procedures involving free flaps and skin grafting and more adjuvant radiotherapy; amputation

also may be required. Unplanned excisions of high-grade soft-tissue sarcomas result in

increased rates of local recurrence but do not diminish disease-specific survival. Patients with

unplanned excisions of high-grade sarcomas require adequate surgical excision and radiation.

Radiation alone or physical therapy is incomplete treatment in this setting. Soft-tissue

sarcomas are often mistaken for spontaneous hematomas because of central tumor necrosis

and intratumoral bleeding, but a hematologic evaluation in a patient with a soft-tissue

sarcoma is not warranted.

RECOMMENDED READINGS

Potter BK, Adams SC, Pitcher JD Jr, Temple HT. Local recurrence of disease after unplanned excisions of high-grade soft tissue sarcomas.

Clin Orthop Relat Res. 2008 Dec;466(12):3093-100. doi: 10.1007/s11999-008-0529-4. Epub 2008 Sep 26. PubMed PMID: 18818981.

View Abstract at PubMed

Temple HT, Worman DS, Mnaymneh WA. Unplanned surgical excision of tumors of the foot and ankle. Cancer Control. 2001 May-

Jun;8(3):262-8. PubMed PMID: 11378652. View Abstract at PubMed

Figure 83a

Figure 83b

Figure 83c

Question 83 of 143

Figures 83a through 83c are the radiograph and MRI scans of a 16-year-old girl who had

posterior knee pain after a dance recital 3 weeks ago; the pain resolved 1 week ago with

ibuprofen use. What is the appropriate treatment for this patient?

Page 60: 2014 Musculoskeletal Tumors and Diseases

1- Biopsy and resection of lesion

2- Observation and serial radiographs

3- Tc-99 whole-body bone scan and fine-cut CT scan

4- Evaluation by a pediatric oncologist

PREFERRED RESPONSE: 2- Observation and serial radiographs

DISCUSSION

The images reveal a small reactive-type lesion in the posteromedial aspect of the distal femur

consistent with an avulsive cortical irregularity. Also referred to as a cortical desmoid,

periosteal desmoid, or “tug lesion,” it is seen most commonly in young adolescents, with a

slight preponderance in boys, with one-third occurring bilaterally. It is thought to be related

to repeated microtrauma with pulling of the adductor magnus or medial gastrocnemius head.

Proper treatment involves recognition of this benign disorder without further work-up. Often

best seen on an oblique radiograph, the lack of soft-tissue mass or bone destruction leads to

the benign diagnosis. Serial radiographs may show complete resolution by age 20.

RECOMMENDED READINGS

Gould CF, Ly JQ, Lattin GE Jr, Beall DP, Sutcliffe JB 3rd. Bone tumor mimics: avoiding misdiagnosis. Curr Probl Diagn Radiol. 2007

May-Jun;36(3):124-41. Review. PubMed PMID: 17484955. View Abstract at PubMed

Yamazaki T, Maruoka S, Takahashi S, Saito H, Takase K, Nakamura M, Sakamoto K. MR findings of avulsive cortical irregularity of the distal femur. Skeletal Radiol. 1995 Jan;24(1):43-6. PubMed PMID: 7709251.

View Abstract at PubMed

Damron TA, Morris C, Rougraff B, Tamurian R. Diagnosis and treatment of joint-related tumors that mimic sports-related injuries. Instr

Course Lect. 2009;58:833-47. PubMed PMID: 19385590. View Abstract at PubMed

Figure 84a

Figure 84b

CLINICAL SITUATION FOR QUESTIONS 84 THROUGH 86

Figures 84a and 84b are the CT and MRI scans of a 17-year-old girl with a painful

lumbosacral scoliosis that has been present for 12 months. Examination is notable only for

pain over the left sacral region and a postural scoliosis leaning away from this side.

Question 84 of 143

The girl’s parents should be advised that their daughter’s condition

1- will respond to bracing and require treatment if curve magnitude exceeds 45 degrees at

skeletal maturity.

2- results from a genetic mutation.

Page 61: 2014 Musculoskeletal Tumors and Diseases

3- requires prolonged antibiotic treatment.

4- is likely to respond to percutaneous intervention.

PREFERRED RESPONSE: 4- is likely to respond to percutaneous intervention.

Question 85 of 143

Medical treatment for this condition

1- uses nonsteroidal anti-inflammatory or aspirin medications.

2- uses radioactive iodine as first-line treatment.

3- centers on antimicrobial susceptibilities.

4- requires autologous stem cell transplantation.

PREFERRED RESPONSE: 1- uses nonsteroidal anti-inflammatory or aspirin medications.

Question 86 of 143

With treatment, the spinal deformity is expected to

1- spontaneously resolve.

2- remain stable and nonprogressive.

3- respond in proportion to the family’s compliance with brace treatment.

4- resolve in the coronal plane and progress in the sagittal plane.

PREFERRED RESPONSE: 1- spontaneously resolve.

DISCUSSION

This young woman has classic imaging features of a sacral ala osteoid osteoma. Preferred

treatment is removal or ablation of the nidus by surgery or percutaneous techniques. Bracing

may be used for adolescent idiopathic scoliosis, but plays no role in the treatment of scoliosis

secondary to osteoid osteoma. Antibiotics would be used for osteomyelitis. There is no

known genetic predisposition to osteoid osteoma.

Nonsteroidal medications or aspirin will alleviate symptoms in the short term. Antibiotics,

radioactive iodine, and stem cell transplantation play no role in treatment for osteoid

osteoma.

The “scoliosis” is expected to resolve following treatment. Bracing is not needed. No lasting

spinal deformity is expected.

RECOMMENDED READINGS

Atesok KI, Alman BA, Schemitsch EH, Peyser A, Mankin H. Osteoid osteoma and osteoblastoma. J Am Acad Orthop Surg. 2011 Nov;19(11):678-89. Review. PubMed PMID: 22052644.

View Abstract at PubMed

Page 62: 2014 Musculoskeletal Tumors and Diseases

Kelly CM. Benign tumors of bone. In: Damron TA, ed. Orthopedic Surgery Essentials: Oncology and Basic Science. Philadelphia, PA:

Lippincott Williams and Wilkins; 2008:54-60.

Figure 87a

Figure 87b

Figure 87c

Figure 87d

Figure 87e

Question 87 of 143

Figures 87a through 87e are the radiograph, MRI scans, and biopsy specimen of an 83-year-

old woman who is experiencing pain in her distal thigh with activity and at night. She has

undergone total hip arthroplasty for hip osteoarthritis. The most appropriate treatment is

1- external beam radiation.

2- curetting and cementation.

3- radiofrequency ablation.

4- wide local resection.

PREFERRED RESPONSE: 4- wide local resection.

DISCUSSION

The imaging shows a cartilaginous lesion with destruction and a malignant extraosseous mass

most consistent with a dedifferentiated chondrosarcoma. The histology demonstrates a high-

grade spindle-cell sarcoma abutting a cartilaginous tumor, confirming this diagnosis.

Treatment in this patient would be wide resection. The role of chemotherapy in this tumor is

unproven, and an 85-year-old patient is an unlikely candidate for such treatment.

Radiofrequency ablation, curettage, and radiation do not play a role in the treatment of this

tumor.

RECOMMENDED READINGS

Mavrogenis AF, Ruggieri P, Mercuri M, Papagelopoulos PJ. Dedifferentiated chondrosarcoma revisited. J Surg Orthop Adv. 2011

Summer;20(2):106-11. Review. PubMed PMID: 21838071.

View Abstract at PubMed

Dickey ID, Rose PS, Fuchs B, Wold LE, Okuno SH, Sim FH, Scully SP. Dedifferentiated chondrosarcoma: the role of chemotherapy with

updated outcomes. J Bone Joint Surg Am. 2004 Nov;86-A(11):2412-8. PubMed PMID: 15523011. View Abstract at PubMed

Page 63: 2014 Musculoskeletal Tumors and Diseases

Figure 88a

Figure 88b

Figure 88c

CLINICAL SITUATION FOR QUESTIONS 88 through 92

Figures 88a and 88b are the radiographs of a 70-year-old woman with a remote history of

localized renal cell carcinoma. She has insidious onset of right shoulder pain that worsens

with any activity and at night. She appears otherwise healthy.

Question 88 of 143

The next treatment step is

1- radiotherapy.

2- CT scan of the chest, abdomen, and pelvis.

3- bone marrow biopsy.

4- surgical stabilization.

PREFERRED RESPONSE: 2- CT scan of the chest, abdomen, and pelvis.

Question 89 of 143

What is the role of presurgical lesional biopsy for this patient?

1- Unnecessary, considering the known history of renal cell carcinoma

2- Should be performed before any surgical treatment

3- Should be performed if the lesion is solitary, but not necessarily if multifocal

4- Should always be accompanied by bone marrow biopsy

PREFERRED RESPONSE: 3- Should be performed if the lesion is solitary, but not

necessarily if multifocal

Question 90 of 143

Representative histology is shown in Figure 88c. Advanced imaging shows mild extraosseous

extent of disease, nondisplaced pathologic fracture, and no neurovascular encasement. The

humerus is best managed with

Page 64: 2014 Musculoskeletal Tumors and Diseases

1- radiotherapy.

2- closed intramedullary rodding.

3- curettage, cancellous allograft, and plate fixation.

4- resection and reconstruction.

PREFERRED RESPONSE: 4- resection and reconstruction.

Question 91 of 143

Chemotherapeutic treatment for this patient

1- centers on bone marrow transplantation.

2- has severe myelosuppressive effects.

3- inhibits vascular endothelial growth factor (VEGF) pathways.

4- is given as a radiation sensitizer.

PREFERRED RESPONSE: 3- inhibits vascular endothelial growth factor (VEGF) pathways.

Question 92 of 143

Oncologic outcome for this patient

1- is influenced by extraosseous tumor extension.

2- is more favorable if the lesion is solitary.

3- involves a predictable rapid demise.

4- hinges on the presence or absence of gene amplification.

PREFERRED RESPONSE: 2- is more favorable if the lesion is solitary.

DISCUSSION

This patient has a lytic lesion filling the proximal humerus with a history of renal cell

carcinoma. In a patient with a history of localized cancer with a bone lesion, staging with CT

scan of the chest, abdomen, and pelvis (and a bone scan, as well) is indicated to define the

extent of disease. A positron emission tomography scan, not given as an option, would be a

reasonable substitute for a CT and bone scan. The most likely clinical scenario is the

development of metastatic renal cell carcinoma, and these tests will define the extent of

disease. Immediate radiotherapy would not be indicated without a diagnosis; as well, renal

cell carcinoma’s response to radiotherapy is unpredictable, and the extent of bony destruction

implies a degree of mechanical insufficiency that would benefit from surgical treatment.

Bone marrow biopsy would be indicated for hematopoietic tumors such as myeloma or

lymphoma. Reflexive surgery should be avoided until a diagnosis is obtained or a widespread

metastatic process is identified to avoid inadvertently mistreating a new sarcoma. As well,

curettage procedures near joints are not very durable in renal cell carcinoma because of its

aggressive behavior and radioresistance.

Page 65: 2014 Musculoskeletal Tumors and Diseases

Biopsy generally is not necessary for patients with established metastatic disease and typical

presentations prior to surgical management. Additionally, if a patient has new metastatic

disease and multiple lesions, biopsy is not needed before surgery because histologic

confirmation can be obtained at surgery. However, for a patient with a solitary lesion and no

history of metastatic disease, biopsy is indicated to exclude a new primary tumor before

surgical treatment is undertaken.

The histology reveals clear-cell carcinoma consistent with a renal primary. Radiotherapy

alone is a poor option considering the unpredictable response of renal cell carcinoma to

radiation and the presence of a pathologic fracture. Closed intramedullary rodding would

provide little proximal fixation. Both this option and curettage are not optimal because of

significant bleeding and lack of durability in this situation. Amputation is seldom indicated

for metastatic disease (even if the neurovascular bundle is encased, which it is not). Formal

resection and reconstruction provides the most durable functional outcome. For a solitary

lesion, resection of the entire lesion would be preferable, although in other situations,

intralesional resection and prosthetic reconstruction may be applicable. If an intralesional

procedure is planned, embolization should be considered before surgery.

Recent understanding of the role of the VEGF pathway in renal cell carcinoma has led to the

use of tyrosine kinase inhibitors and related agents for chemotherapy. These can have wound-

healing effects (critical for surgeons to recognize and understand), but generally are not

myelosuppressive. Objective response rates approach 30%. Bone marrow transplantation and

radiosensitizing chemotherapy are not used in this situation.

Patients with metastatic renal cell carcinoma to bone have a median survival of 11.5 months

in the largest study, and this likely improves with newer adjuvant therapies. Patients with

solitary lesions have longer survival, but the likelihood of true cure is low even with en bloc

resection. There is no known prognostic significance of extraosseous extension or gene

amplification in renal carcinoma.

RECOMMENDED READINGS

Lin PP, Mirza AN, Lewis VO, Cannon CP, Tu SM, Tannir NM, Yasko AW. Patient survival after surgery for osseous metastases from renal cell carcinoma. J Bone Joint Surg Am. 2007 Aug;89(8):1794-801. PubMed PMID: 17671020.

View Abstract at PubMed

Posadas EM, Figlin RA. Systemic therapy in renal cell carcinoma: advancing paradigms. Oncology (Williston Park). 2012 Mar;26(3):290-

301. Review. PubMed PMID: 22545314.

View Abstract at PubMed

Biermann JS, Holt GE, Lewis VO, Schwartz HS, Yaszemski MJ. Metastatic bone disease: diagnosis, evaluation, and treatment. J Bone Joint

Surg Am. 2009 Jun;91(6):1518-30. Review. PubMed PMID: 19487533. View Abstract at PubMed

Figure 93a

Figure 93b

Page 66: 2014 Musculoskeletal Tumors and Diseases

Question 93 of 143

Figures 93a and 93b are the MRI scans of a 24-year-old man with painless, persistent

swelling in his left knee without any trauma. What is the best next treatment step?

1- Arthroscopic anterior synovectomy and posterior open resection

2- Arthroscopic anterior synovectomy only

3- Radiation therapy and wide excision

4- Observation

PREFERRED RESPONSE: 3- Radiation therapy and wide excision

DISCUSSION

The lesion seen in the figures is pigmented villonodular synovitis (PVNS), as shown by the

dark areas on MRI scan attributable to chronic hemosiderin deposition. The age of the

patient, symptom of persistent swelling, and lack of trauma are all consistent with a synovial

proliferative disorder. The fronds arising from the synovium make PVNS the most likely of

the synovial proliferative diseases, which include PVNS, synovial chondromatosis, lipoma

arborescens, and synovial hemangioma. The imaging shows diffuse disease throughout the

knee in the anterior compartment and extra-articular disease extending from the posterior

compartment. The current accepted standard of care for initial presentation and treatment is

an anterior synovectomy and an open posterior approach to resect the extra-articular disease.

Recurrence rates for diffuse disease are higher with arthroscopic compared to open

synovectomy, although morbidity is lower with arthroscopic treatment. Radiation therapy and

wide excision would be reserved for a soft-tissue sarcoma, and, in the setting of a chronic

effusion, observation would not suffice because the joint is at risk for degradation from the

chronic hemosiderin and inflammatory reaction in the intra-articular space.

RECOMMENDED READINGS

van der Heijden L, Gibbons CL, Dijkstra PD, Kroep JR, van Rijswijk CS, Nout RA, Bradley KM, Athanasou NA, Hogendoorn PC, van de

Sande MA. The management of diffuse-type giant cell tumour (pigmented villonodular synovitis) and giant cell tumour of tendon sheath (nodular tenosynovitis). J Bone Joint Surg Br. 2012 Jul;94(7):882-8. doi: 10.1302/0301-620X.94B7.28927. Review. PubMed PMID:

22733940.

View Abstract at PubMed

Masih S, Antebi A. Imaging of pigmented villonodular synovitis. Semin Musculoskelet Radiol. 2003 Sep;7(3):205-16. Review. PubMed

PMID: 14593562. View Abstract at PubMed

Question 94 of 143

What biopsy technique for a posterior thigh sarcoma is associated with the highest risk for

adverse outcome?

1- Transverse incision open biopsy

2- Core needle biopsy

3- Fine-needle aspirate

4- Longitudinal incision open biopsy

Page 67: 2014 Musculoskeletal Tumors and Diseases

PREFERRED RESPONSE: 1- Transverse incision open biopsy

DISCUSSION

Fundamental planning and execution of a biopsy are imperative to maximize a surgeon’s

technical ability to perform limb-salvage surgery and preclude risk for amputation as a result

of a poorly executed biopsy. An incorrectly performed biopsy can place a patient who is a

limb-salvage candidate into a non-limb-salvage category. Transverse incisions for a sarcoma

on an extremity are absolutely contraindicated. A transverse incision increases the extent of

dissection necessary to achieve an appropriate radical resection, contaminates more anatomic

planes, and places neurovascular structures at higher risk for contamination. In many

instances, a free vascularized muscle flap and/or skin grafting is needed to close the defect

created by an incorrectly performed biopsy. Meticulous hemostasis at the time of biopsy is

also essential to avoid contamination of additional tissue, thus expanding the extent of

dissection.

For open biopsies, incisions in line with the long axis of a limb are mandatory. Many tumors

are amenable to core needle biopsy. Placement of the needle or incision must be carefully

thought out to ensure that the biopsy tract may be excised in continuity with the mass at the

time of definitive resection surgery. In an ideal situation, the biopsy should be performed by

the surgeon, who will perform the definitive procedure. This obviates the need to speculate

about needle tracts and biopsy techniques. Fine-needle aspirate biopsies are generally safe,

but likely have lower diagnostic yield.

RECOMMENDED READINGS

Enneking WF. Musculoskeletal Tumor Surgery. Vol 1. Philadelphia, PA: Churchill-Livingstone; 1983: Chapters 1-3, 7.

Enneking WF, Maale GE. The effect of inadvertent tumor contamination of wounds during the surgical resection of musculoskeletal neoplasms. Cancer. 1988 Oct 1;62(7):1251-6. PubMed PMID: 3416267.

View Abstract at PubMed

Heslin MJ, Lewis JJ, Woodruff JM, Brennan MF. Core needle biopsy for diagnosis of extremity soft tissue sarcoma. Ann Surg Oncol. 1997

Jul-Aug;4(5):425-31. PubMed PMID: 9259971.

View Abstract at PubMed

Mankin HJ, Lange TA, Spanier SS. The hazards of biopsy in patients with malignant primary bone and soft-tissue tumors. J Bone Joint Surg

Am. 1982 Oct;64(8):1121-7. PubMed PMID: 7130225. View Abstract at PubMed

Shiu MH, Brennan MF. Surgical Management of Soft Tissue Sarcoma. Philadelphia, PA: Lea and Febiger; 1989:Chapter 6; 58-64.

Question 95 of 143

A 60-year-old woman with a history of breast cancer has a rapidly enlarging arm mass. The

lesion is situated outside of the prior irradiation field, but within an area of heavy

lymphedema involvement. Needle biopsy reveals a high-grade sarcoma. What is the most

likely diagnosis?

1- Lymphangiosarcoma

2- Acral myxoinflammatory fibroblastic sarcoma

Page 68: 2014 Musculoskeletal Tumors and Diseases

3- Hemangioendothelioma

4- Hemangiopericytoma

PREFERRED RESPONSE: 1- Lymphangiosarcoma

DISCUSSION

Lymphangiosarcomas that arise in pre-existing lymphedema are aggressive malignancies (ie,

Stewart-Treves syndrome) that require resection or amputation. Acral myxoinflammatory

fibroblastic sarcoma is a low-grade sarcoma. Hemangioendothelioma and

hemangiopericytoma are not typically associated with chronic lymphedema.

RECOMMENDED READINGS

Hellman S, DeVita VT, Rosenberg S, eds. Cancer: Principles and Practice of Oncology. Philadelphia, PA: Lippincott-Raven; 2001:1853.

Chung KC, Kim HJ, Jeffers LL. Lymphangiosarcoma (Stewart-Treves syndrome) in postmastectomy patients. J Hand Surg Am. 2000

Nov;25(6):1163-8. Review. PubMed PMID: 11119680. View Abstract at PubMed

Figure 96a

Figure 96b

Figure 96c

Figure 96d

RESPONSES FOR QUESTIONS 96 THROUGH 100

1- Osteosarcoma

2- Ewing sarcoma

3- Langerhans cell histiocytosis (LCH)

4- Osteomyelitis

5- Primary lymphoma of bone

Match the condition above to the patient description below.

Question 96 of 143

10-year-old girl with sickle cell anemia and a swollen, erythematous, and painful elbow

(Figures 96a through 96d)

Page 69: 2014 Musculoskeletal Tumors and Diseases

1- Osteosarcoma

2- Ewing sarcoma

3- Langerhans cell histiocytosis (LCH)

4- Osteomyelitis

5- Primary lymphoma of bone

PREFERRED RESPONSE: 4- Osteomyelitis

Figure 97a

Figure 97b

Figure 97c

Figure 97d

Figure 97e

Figure 97f

RESPONSES FOR QUESTIONS 96 THROUGH 100

1- Osteosarcoma

2- Ewing sarcoma

3- Langerhans cell histiocytosis (LCH)

4- Osteomyelitis

5- Primary lymphoma of bone

Match the condition above to the patient description below.

Question 97 of 143

12-year-old girl with knee and lower leg pain and swelling for 2 months after minor trauma

(Figures 97a through 97f)

1- Osteosarcoma

2- Ewing sarcoma

3- Langerhans cell histiocytosis (LCH)

4- Osteomyelitis

5- Primary lymphoma of bone

PREFERRED RESPONSE: 1- Osteosarcoma

Page 70: 2014 Musculoskeletal Tumors and Diseases

Figure 98a

Figure 98b

Figure 98c

Figure 98d

Figure 98e

RESPONSES FOR QUESTIONS 96 THROUGH 100

1- Osteosarcoma

2- Ewing sarcoma

3- Langerhans cell histiocytosis (LCH)

4- Osteomyelitis

5- Primary lymphoma of bone

Match the condition above to the patient description below.

Question 98 of 143

11-year-old boy with severe pain in the lower back and pelvic region, particularly with

activity and at night; CD99 immunostaining positive (Figures 98a through 98e)

1- Osteosarcoma

2- Ewing sarcoma

3- Langerhans cell histiocytosis (LCH)

4- Osteomyelitis

5- Primary lymphoma of bone

PREFERRED RESPONSE: 2- Ewing sarcoma

Figure 99a

Figure 99b

Figure 99c

Figure 99d

Figure 99e

Figure 99f

RESPONSES FOR QUESTIONS 96 THROUGH 100

Page 71: 2014 Musculoskeletal Tumors and Diseases

1- Osteosarcoma

2- Ewing sarcoma

3- Langerhans cell histiocytosis (LCH)

4- Osteomyelitis

5- Primary lymphoma of bone

Match the condition above to the patient description below.

Question 99 of 143

4-year-old boy who began limping after having increasing pain in his lower thigh and knee

for 3 months (Figures 99a through 99f)

1- Osteosarcoma

2- Ewing sarcoma

3- Langerhans cell histiocytosis (LCH)

4- Osteomyelitis

5- Primary lymphoma of bone

PREFERRED RESPONSE: 3- Langerhans cell histiocytosis (LCH)

Figure 100a

Figure 100b

Figure 100c

Figure 100d

Figure 100e

Figure 100f

RESPONSES FOR QUESTIONS 96 THROUGH 100

1- Osteosarcoma

2- Ewing sarcoma

3- Langerhans cell histiocytosis (LCH)

4- Osteomyelitis

5- Primary lymphoma of bone

Match the condition above to the patient description below.

Page 72: 2014 Musculoskeletal Tumors and Diseases

Question 100 of 143

22-year-old man with rapidly growing mass and severe swelling in the knee; CD20-positive

on immunostaining (Figures 100a through 100f)

1- Osteosarcoma

2- Ewing sarcoma

3- Langerhans cell histiocytosis (LCH)

4- Osteomyelitis

5- Primary lymphoma of bone

PREFERRED RESPONSE: 5- Primary lymphoma of bone

DISCUSSION

Osteomyelitis in the pediatric population occurs in approximately 1 in 5000 children.

Staphylococcus aureus accounts for approximately 80% of all infections. Staphylococcus

remains the most common organism in patients with sickle cell anemia; however, an unusual

culprit in these patients can be salmonella. Imaging studies may not be specific but often

show periosteal reaction, variable bone loss and production, and significant inflammation on

MRI scan. The classic sign on radiograph is “bone-in-bone” for subacute and chronic cases.

This often signifies new bone (involucrum) surrounding the necrotic “trapped” sequestrum.

The histopathology varies depending on the acuity of disease. A variety of inflammatory cells

often will be seen. Typically, more neutrophils are seen in acute disease, and lymphocytes

and macrophages in chronic disease. Chronic cases normally show focal areas of bone

necrosis with empty lacunae.

Osteosarcoma is the most common primary cancer of bone excluding multiple myeloma. It

most often occurs during the first 2 decades of life. The imaging studies show an aggressive

bone loss and new bone formation process in addition to a soft-tissue mass on MRI scan. In

this case, the radiograph shows an excellent example of a “sunburst” or “hair-on-end” pattern

signifying the rapid growth of the tumor and subsequent periosteal reaction. The

histopathology varies by subtype; however, the most common variety (classic, high grade) as

in this scenario demonstrates hypercellular round or spindle cells with atypia, pleomorphism,

atypical mitoses, and osteoid production. Ewing sarcoma is the second-most-common bone

sarcoma in children. It will demonstrate aggressive changes on radiograph, but, unlike

osteosarcoma, it frequently does not produce new bone within the matrix of the tumor.

However, periosteal elevation and bone reaction are often seen. This case shows subtle

changes on radiograph but impressive changes on MRI scan with a soft-tissue mass and

extensive bone marrow involvement. This is fairly common for small round blue-cell tumors

such as Ewing sarcoma/primitive neuroectodermal tumor, lymphoma, multiple myeloma, and

others. The histopathology often shows hypercellularity with sheets of small round blue cells.

In most cases, CD99 is positive and a characteristic 11:22 translocation is seen, making the

diagnosis definitive.

LCH is a rare condition with an uncertain pathogenesis. It is generally seen in patients

younger than 20 years of age and often can mimic other conditions such as infection or

cancer. The imaging studies can range from benign-looking with a focal, well-circumscribed

lesion to more common aggressive lesions with less-defined borders, periosteal reaction,

associated edema, and, in some cases, pathologic fracture. The histopathology in this case is

Page 73: 2014 Musculoskeletal Tumors and Diseases

critical to the diagnosis. The presence of Langerhans cells in a granulomatous-like setting

with a reactive background of macrophages, eosinophils, multinucleate giant cells, and T

cells is important to identify. The Langerhans cell is a large, ovoid, mononuclear cell with a

folded nucleus, a discrete nucleolus, and moderate amounts of slightly eosinophilic

homogeneous cytoplasm. On electron microscopy, Birbeck granules with their tennis racquet

form can be seen in Langerhans cells. Primary lymphoma of bone is rare. It accounts for

approximately 1% of all lymphoma cases. As with other small round blue-cell tumors, the

imaging may vary. Osteolytic and osteoblastic findings may be present on radiograph, but

often the MRI scan reveals a larger-than-expected soft-tissue component. The histopathology

shows sheets of small round blue cells. These can be differentiated from others in this

category by flow cytometry and also by using specialized surface markers like CD20, as in

this case.

RECOMMENDED READINGS

Thomsen I, Creech CB. Advances in the diagnosis and management of pediatric osteomyelitis. Curr Infect Dis Rep. 2011 Oct;13(5):451-60.

doi: 10.1007/s11908-011-0202-z. PubMed PMID: 21789499. View Abstract at PubMed

Hatzenbuehler J, Pulling TJ. Diagnosis and management of osteomyelitis. Am Fam Physician. 2011 Nov 1;84(9):1027-33. Review. PubMed PMID: 22046943.

View Abstract at PubMed

Ejindu VC, Hine AL, Mashayekhi M, Shorvon PJ, Misra RR. Musculoskeletal manifestations of sickle cell disease. Radiographics. 2007

Jul-Aug;27(4):1005-21. Review. PubMed PMID: 17620464. View Abstract at PubMed

Jaffe N. Osteosarcoma: review of the past, impact on the future. The American experience. Cancer Treat Res. 2009;152:239-62. doi: 10.1007/978-1-4419-0284-9_12. Review. PubMed PMID: 20213394.

View Abstract at PubMed

Messerschmitt PJ, Garcia RM, Abdul-Karim FW, Greenfield EM, Getty PJ. Osteosarcoma. J Am Acad Orthop Surg. 2009 Aug;17(8):515-

27. Review. PubMed PMID: 19652033.

View Abstract at PubMed

Maheshwari AV, Cheng EY. Ewing sarcoma family of tumors. J Am Acad Orthop Surg. 2010 Feb;18(2):94-107. Review. PubMed PMID:

20118326. View Abstract at PubMed

Abla O, Egeler RM, Weitzman S. Langerhans cell histiocytosis: Current concepts and treatments. Cancer Treat Rev. 2010 Jun;36(4):354-9. doi: 10.1016/j.ctrv.2010.02.012. Epub 2010 Feb 25. Review. PubMed PMID: 20188480.

View Abstract at PubMed

Mikhaeel NG. Primary bone lymphoma. Clin Oncol (R Coll Radiol). 2012 Jun;24(5):366-70. doi: 10.1016/j.clon.2012.02.006. Epub 2012

Mar 6. Review. PubMed PMID: 22402011.

View Abstract at PubMed

Figure

101a

Figure

101b

Figure

102a

Figure

102b

Figure

103a

Figure

103b

Figure

104a

Figure

104b

RESPONSES FOR QUESTIONS 101 THROUGH 104

Page 74: 2014 Musculoskeletal Tumors and Diseases

1- Clear cell renal carcinoma

2- Squamous carcinoma of the lung

3- Multiple myeloma

4- Metastatic adenocarcinoma

Match each set of photomicrographs with the correct diagnosis.

Question 101 of 143

Figures 101a and 101b

1- Clear cell renal carcinoma

2- Squamous carcinoma of the lung

3- Multiple myeloma

4- Metastatic adenocarcinoma

PREFERRED RESPONSE: 1- Clear cell renal carcinoma

Question 102 of 143

Figures 102a and 102b

1- Clear cell renal carcinoma

2- Squamous carcinoma of the lung

3- Multiple myeloma

4- Metastatic adenocarcinoma

PREFERRED RESPONSE: 4- Metastatic adenocarcinoma

Question 103 of 143

Figures 103a and 103b

1- Clear cell renal carcinoma

2- Squamous carcinoma of the lung

3- Multiple myeloma

4- Metastatic adenocarcinoma

PREFERRED RESPONSE: 2- Squamous carcinoma of the lung

Question 104 of 143

Figures 104a and 104b

Page 75: 2014 Musculoskeletal Tumors and Diseases

1- Clear cell renal carcinoma

2- Squamous carcinoma of the lung

3- Multiple myeloma

4- Metastatic adenocarcinoma

PREFERRED RESPONSE: 3- Multiple myeloma

DISCUSSION

These photomicrographs represent metastatic entities commonly found in the bone. Renal

carcinoma is distinguished by large cells with vacuolated cytoplasm that warrant the term

"clear cell" carcinoma. Squamous carcinomas are characterized by nests of epithelial cells

that are relatively monotonous and form layers or sheets of cells. Myeloma is a plasma cell

proliferative disorder, and the cells are classified by the "clock-face" nuclei and basophilic

cytoplasm. Adenocarcinomas are noted to have the appearance of malignant cells forming

gland-like structures in the invaded tissue. The more well-differentiated the tumor, the more

prominent and distinctive are the glands. These representations are relatively classic for

lesions such as prostate and breast adenocarcinoma.

RECOMMENDED READINGS

Layfield LJ. Cytopathology of Bone and Soft Tissue Tumors. Oxford, England: Oxford University Press; 2002.

Unni KK, Inwards CY. Dahlin's Bone Tumors. 6th ed. Philadelphia, PA: Lippincott Williams & Wilkins; 2009.

Figure 105a

Figure 105b

Figure 105c

Figure 105d

Figure 105e

Figure 105f

Question 105 of 143

A 22-year-old mother noticed left leg pain while lifting her only child. A previous biopsy of

her left tibia 7 years ago had been interpreted as osteofibrous dysplasia. She has no other

medical history. Her current anteroposterior and lateral radiographs are shown in Figures

105a and 105b. Her comparative T2-weighted axial MRI scan from a similar area of the

diaphyseal tibia is shown in Figure 105c. The histologic specimens are shown in Figures

105d and 105e. A cytokeratin stain is shown in Figure 105f. The diagnosis is

1- osteofibrous dysplasia.

2- osteofibrous dysplasia-like adamantinoma.

3- low-grade osteosarcoma.

4- metastatic adenocarcinoma.

Page 76: 2014 Musculoskeletal Tumors and Diseases

PREFERRED RESPONSE: 2- osteofibrous dysplasia-like adamantinoma.

DISCUSSION

An insidious onset of pain is a characteristic symptom in patients with adamantinoma. The

characteristic radiographic finding is a “soap-bubble” osteolytic lesion of the tibia with

surrounding sclerosis. A soft-tissue mass also is characteristic. Few keratin-positive cells are

seen in osteofibrous dysplasia. The characteristic fibrous stroma with spicules of bone with

basophilic epithelioid cells is diagnostic of adamantinoma. The other diagnoses mentioned do

not have these characteristics.

RECOMMENDED READINGS

Qureshi AA, Shott S, Mallin BA, Gitelis S. Current trends in the management of adamantinoma of long bones. An international study. J

Bone Joint Surg Am. 2000 Aug;82-A(8):1122-31. PubMed PMID: 10954102.

View Abstract at PubMed

Most MJ, Sim FH, Inwards CY. Osteofibrous dysplasia and adamantinoma. J Am Acad Orthop Surg. 2010 Jun;18(6):358-66. Review.

PubMed PMID: 20511441. View Abstract at PubMed

Figure 106a

Figure 106b

Figure 106c

Question 106 of 143

Figures 106a and 106b are the T1 sagittal and T2 coronal images of a 41-year-old woman

who has been experiencing electric shock-type radiating pain over her right ankle for 4½

years. Her biopsy specimen is shown in Figure 106c. What is the best next treatment step?

1- Marginal excision

2- Wide excision

3- Wide excision and radiation

4- Observation

PREFERRED RESPONSE: 1- Marginal excision

DISCUSSION

The lesion seen here is a benign nerve sheath tumor—specifically, a schwannoma. The

diagnosis of neurogenic tumors can be suggested from their imaging appearance, including

lesion shape and intrinsic imaging characteristics such as target and string signs. The

histology shows benign nerve elements including verocay bodies with Antoni A (more

cellular) and Antoni B (less cellular) areas. Most patients with benign nerve sheath tumors

Page 77: 2014 Musculoskeletal Tumors and Diseases

present with a painful mass, paresthesias, or numbness without significant weakness.

Marginal excision with preservation of the associated nerve fibers is the optimal treatment for

symptomatic schwannomas. Wide excision eliminates symptoms successfully with minimal

concern for recurrence, but should be avoided to preserve nerve function. Wide excision with

or without radiation is not warranted in this benign condition, and, although observation is

acceptable for difficult locations or asymptomatic Schwannomas, the persistent symptoms

experienced by this patient preclude observation as the best option.

RECOMMENDED READINGS

Murphey MD, Smith WS, Smith SE, Kransdorf MJ, Temple HT. From the archives of the AFIP. Imaging of musculoskeletal neurogenic

tumors: radiologic-pathologic correlation. Radiographics. 1999 Sep-Oct;19(5):1253-80. PubMed PMID: 10489179.

View Abstract at PubMed

Levi AD, Ross AL, Cuartas E, Qadir R, Temple HT. The surgical management of symptomatic peripheral nerve sheath tumors. Neurosurgery. 2010 Apr;66(4):833-40. doi: 10.1227/01.NEU.0000367636.91555.70. PubMed PMID: 20190660.

View Abstract at PubMed

Figure 107a

Figure 107b

Figure 107c

Question 107 of 143

Figures 107a through 107c are the radiograph, CT, and bone scan of a 68-year-old man.

While walking, he collapsed and was unable to ambulate because of pain and deformity in his

right leg. What is the most appropriate next step?

1- Staging studies to assess the extent of disease

2- Immediate stabilization of the fracture with an intramedullary nail

Page 78: 2014 Musculoskeletal Tumors and Diseases

3- Curetting and bone culture

4- Segmental resection of the tibia and allograft reconstruction

PREFERRED RESPONSE: 1- Staging studies to assess the extent of disease

DISCUSSION

This patient has a pathologic fracture; in patients older than age 50, the most likely diagnosis

is metastatic carcinoma or hematopoietic malignancy. A systemic work-up with staging

imaging studies and laboratory tests as outlined by Rougraff and associates will identify the

primary site of disease in approximately 90% of patients. A biopsy may not be necessary, and

this protocol will also effectively exclude a primary sarcoma in the vast majority of patients.

The other options all initiate treatment without first evaluating for extent of disease and

diagnosis. If this turns out to be a sarcoma, stabilization or curetting the lesion might make

subsequent limb salvage difficult or impossible. Segmental resection adds unnecessary

morbidity to the surgical procedure in the more common circumstance that this is revealed to

be metastatic carcinoma.

RECOMMENDED READINGS

Biermann JS, Holt GE, Lewis VO, Schwartz HS, Yaszemski MJ. Metastatic bone disease: diagnosis, evaluation, and treatment. J Bone Joint

Surg Am. 2009 Jun;91(6):1518-30. Review. PubMed PMID: 19487533. View Abstract at PubMed

Rougraff BT, Kneisl JS, Simon MA. Skeletal metastases of unknown origin. A prospective study of a diagnostic strategy. J Bone Joint Surg Am. 1993 Sep;75(9):1276-81. PubMed PMID: 8408149.

View Abstract at PubMed

RESPONSES FOR QUESTIONS 108 THROUGH 111

1- Glomus tumor

2- Diffuse pigmented villonodular synovitis (PVNS)

3- Extra-abdominal fibromatosis

4- Schwannoma

5- Hemangioma

Please match the characteristics below to the condition listed above.

Question 108 of 143

Painful lesion that frequently is triggered by cold and has characteristics of vascular origin

1- Glomus tumor

2- Diffuse pigmented villonodular synovitis (PVNS)

3- Extra-abdominal fibromatosis

4- Schwannoma

5- Hemangioma

PREFERRED RESPONSE: 1- Glomus tumor

Page 79: 2014 Musculoskeletal Tumors and Diseases

Question 109 of 143

High local recurrence rate despite surgical excision and controversial to treat with

chemotherapy and/or external beam radiation; may have similar appearance as a soft-tissue

sarcoma.

1- Glomus tumor

2- Diffuse pigmented villonodular synovitis (PVNS)

3- Extra-abdominal fibromatosis

4- Schwannoma

5- Hemangioma

PREFERRED RESPONSE: 3- Extra-abdominal fibromatosis

Question 110 of 143

S-100-positive and low recurrence rate after surgical excision

1- Glomus tumor

2- Diffuse pigmented villonodular synovitis (PVNS)

3- Extra-abdominal fibromatosis

4- Schwannoma

5- Hemangioma

PREFERRED RESPONSE: 4- Schwannoma

Question 111 of 143

Most common soft-tissue tumor in childhood and infancy; may fluctuate in size with activity

1- Glomus tumor

2- Diffuse pigmented villonodular synovitis (PVNS)

3- Extra-abdominal fibromatosis

4- Schwannoma

5- Hemangioma

PREFERRED RESPONSE: 5- Hemangioma

DISCUSSION

Hemangiomas are common soft-tissue tumors that resemble the architecture of normal blood

vessels. They are the most common tumors seen during infancy and childhood and they can

be found in any tissue type. Larger, deep hemangiomas commonly cause pain with activity.

They may fluctuate in size and, if superficial enough, skin color changes may be noted.

Page 80: 2014 Musculoskeletal Tumors and Diseases

PVNS is a locally aggressive proliferative disorder of the synovium. It can be found in the

joint, tendon sheath, or synovial bursa. It generally is found in 2 forms: localized or nodular

diffuse or villonodular. The diffuse form has a high local recurrence rate between 33% and

50%. The peak incidence is noted in people who are 20 to 50 years of age and it is more

common in women. Treatment is generally surgical, although radiation implants have been

used for recalcitrant cases.

A schwannoma (neurilemoma) is a common encapsulated nerve sheath tumor. Histologically,

it has 2 components: Antoni A (cellular regions) and Antoni B (loose myxoid regions). Like

most neural tumors, S-100 immunostain is strongly positive. As opposed to neurofibromas,

which are intraneural and seen commonly in patients with neurofibromatosis type I, these

lesions are located within the nerve sheath, more commonly occur as a solitary tumor, and are

generally easier to remove than neurofibromas. However, there is risk for nerve damage with

excision of any nerve sheath tumor, and for difficult locations or asymptomatic

Schwannomas, observation may be appropriate. The local recurrence rate is low with

marginal excision.

Extra-abdominal fibromatosis (desmoid tumor) is locally aggressive and can be found

superficially or deep. The deep variant tends to grow larger and faster. Local recurrence rates

can be high, but vary greatly on location, size, and histology, among other factors. Surgical

excision is the treatment of choice. External beam radiation and chemotherapy have been

used in recurrent disease or to treat regions that are unresectable. In some cases, fibromatoses

can mimic sarcomas, but these tumors do not have metastatic potential.

Glomus tumors are small, painful soft-tissue masses of smooth muscle origin typically in a

subungual location. They often cause moderate to severe pain from touch or cold.

RECOMMENDED READINGS

Bruder E, Alaggio R, Kozakewich HP, Jundt G, Dehner LP, Coffin CM. Vascular and perivascular lesions of skin and soft tissues in children and adolescents. Pediatr Dev Pathol. 2012;15(1 Suppl):26-61. doi: 10.2350/11-11-1119-PB.1. Review. PubMed PMID: 22420724.

View Abstract at PubMed

Coffin CM, Dehner LP. Pathologic evaluation of pediatric soft tissue tumors. Am J Clin Pathol. 1998 Apr;109(4 Suppl 1):S38-52. Review.

PubMed PMID: 9533748.

View Abstract at PubMed

Sheth S, Lai CK, Dry S, Binder S, Fishbein MC. Benign vascular tumors and tumor-like proliferations. Semin Diagn Pathol. 2008

Feb;25(1):1-16. Review. PubMed PMID: 18350917. View Abstract at PubMed

Mendenhall WM, Mendenhall CM, Reith JD, Scarborough MT, Gibbs CP, Mendenhall NP. Pigmented villonodular synovitis. Am J Clin Oncol. 2006 Dec;29(6):548-50. Review. PubMed PMID: 17148989.

View Abstract at PubMed

Molloy AP, Hutchinson B, O'Toole GC. Extra-abdominal desmoid tumours: a review of the literature. Sarcoma. 2012;2012:578052. doi:

10.1155/2012/578052. Epub 2012 Aug 16. PubMed PMID: 22966217.

View Abstract at PubMed

Rutenberg MS, Indelicato DJ, Knapik JA, Lagmay JP, Morris C, Zlotecki RA, Scarborough MT, Gibbs CP, Marcus RB. External-beam

radiotherapy for pediatric and young adult desmoid tumors. Pediatr Blood Cancer. 2011 Sep;57(3):435-42. doi: 10.1002/pbc.22916. Epub

2010 Dec 15. PubMed PMID: 21744472.

View Abstract at PubMed

Gombos Z, Zhang PJ. Glomus tumor. Arch Pathol Lab Med. 2008 Sep;132(9):1448-52. doi: 10.1043/1543-

2165(2008)132[1448:GT]2.0.CO;2. Review. PubMed PMID: 18788860.

View Abstract at PubMed

Page 81: 2014 Musculoskeletal Tumors and Diseases

Figure 112a

Figure 112b

Figure 112c

Figure 112d

Figure 112e

Figure 112f

Question 112 of 143

Figures 112a and 112b are the anteroposterior and lateral radiographs of a 65-year-old man

who has a significant history of tobacco abuse and a 6-week history of right thigh pain. Axial

and sagittal MRI scans are seen in Figures 112c and 112d. His MR angiogram is shown in

Figure 112e. A biopsy of a lesion is shown in Figure 112f. What is the most likely diagnosis?

1- Secondary sarcoma in a pre-existing condition

2- Angiosarcoma

3- Metastatic lung carcinoma

4- Fibrous dysplasia

PREFERRED RESPONSE: 1- Secondary sarcoma in a pre-existing condition

DISCUSSION

The figures show an aggressive destructive lesion in a bone that is already involved with

Paget disease, and the pathology shows highly malignant cells. This represents a secondary

sarcoma arising in Pagetoid bone. Secondary sarcomas occurring in Pagetic bone include

malignant fibrous histiocytoma and chondrosarcomas, but osteosarcomas are the most

common malignancy in these patients. Most patients are older than age 60 and have new pain

in the setting of known Paget disease. The prognosis is generally poor because this is a very

aggressive sarcoma that is difficult to treat in an older population. Wide surgical removal and

chemotherapy are most commonly employed. The pathology does not show rudimentary

vessels or channels which are expected with angiosarcoma, effectively excluding that

diagnosis. A lack of nests or gland formation excludes metastatic lung carcinoma, and the

absence of fibrous stroma intermixed with bone islands or “Chinese characters” excludes

fibrous dysplasia.

RECOMMENDED READINGS

Deyrup AT, Montag AG, Inwards CY, Xu Z, Swee RG, Krishnan Unni K. Sarcomas arising in Paget disease of bone: a clinicopathologic

analysis of 70 cases. Arch Pathol Lab Med. 2007 Jun;131(6):942-6. PubMed PMID: 17550323.

View Abstract at PubMed

Mangham DC, Davie MW, Grimer RJ. Sarcoma arising in Paget's disease of bone: declining incidence and increasing age at presentation.

Bone. 2009 Mar;44(3):431-6. doi: 10.1016/j.bone.2008.11.002. Epub 2008 Nov 19. PubMed PMID: 19064007. View Abstract at PubMed

Page 82: 2014 Musculoskeletal Tumors and Diseases

Figure 113a

Figure 113b

CLINICAL SITUATION FOR QUESTIONS 113 THROUGH 116

Figures 113a and 113b are the radiographs of a 68-year-old-man who has increasing pain in

his left groin with weight-bearing activities and a Trendelenburg gait. Radiographs reveal a

lytic lesion of the greater trochanter. An initial diagnosis of adenocarcinoma of the lung was

made 1 year before this presentation. His lung cancer treatment consisted of partial

lobectomy and postsurgical radiation therapy.

Question 113 of 143

What is the preferred study to stage his disease at this time?

1- Whole-body bone scan

2- Chest/abdomen/pelvis CT scan

3- Bronchoscopy

4- Positron emission tomography (PET) scan

PREFERRED RESPONSE: 4- Positron emission tomography (PET) scan

Question 114 of 143

If this is a solitary metastasis, what is the preferred treatment?

1- Radiation therapy

2- Chemotherapy

3- Curettage, cementation, and internal fixation

4- Resection and prosthetic reconstruction

PREFERRED RESPONSE: 3- Curettage, cementation, and internal fixation

Question 115 of 143

Before receiving treatment, the patient falls and sustains a fracture through the base of the

femoral neck region; the best treatment now is

Page 83: 2014 Musculoskeletal Tumors and Diseases

1- radiation therapy.

2- chemotherapy.

3- curettage, cementation, and internal fixation.

4- resection and prosthetic reconstruction.

PREFERRED RESPONSE: 4- resection and prosthetic reconstruction.

Question 116 of 143

Staging studies show no other lesions and surgical treatment is planned; when should a

biopsy be performed?

1- Before surgery

2- Intraoperatively after instrumentation

3- After surgery (reamings/curettings sent)

4- No biopsy is needed

PREFERRED RESPONSE: 1- Before surgery

DISCUSSION

Patients with a metastatic lung carcinoma are best staged and imaged with a PET scan.

Although many tumors are clinically evaluated with PET scans, metastatic lung cancer is one

of the few “approved” indications for PET imaging. This imaging modality is sensitive for

delineating the extent of disease and helpful for identifying areas that may also be at risk for

pathologic fracture. Stabilization of painful, lytic lesions in weight-bearing bones is

recommended to prevent pathologic fracture. Stabilization of an impending pathologic

fracture is generally easier than stabilization of a pathologic fracture; once the patient in this

vignette sustains a fracture, resection and reconstruction is indicated because stabilization

alone is unlikely to heal and poses high risk for failure. In cases of extreme bone loss,

especially in the epiphyseal region, reconstruction with tumor megaprostheses is desirable to

avoid mechanical failure of additional internal fixation. Patients with widely metastatic

disease may not be able to use ambulatory assistance aids effectively, placing traditional

internal fixation at risk for failure. Use of megaprostheses obviates the need for adjuvant

therapies to correct the loss of bone stock.

Radiation therapy alone to lesions that are radiosensitive is effective but does not alleviate the

risk for pathologic fracture. Chemotherapy alone is insufficient treatment for bone

metastases. Hormonal therapy is acceptable for treatment of hormone-sensitive tumors;

however, is not sufficient treatment for an impending pathologic fracture.

For patients with remote diagnoses of carcinoma, biopsy of new-onset presumed metastases

is mandatory to avoid an incorrect treatment regimen being employed. Patients can develop a

secondary malignancy or infection, both of which will require a different treatment regimen.

Biopsy should be done and diagnosis established either preoperatively or intraoperatively

prior to instrumentation (with frozen section analysis) to avoid contamination of the femoral

canal and soft-tissue envelope. Sending a specimen after instrumenting the femur or after

Page 84: 2014 Musculoskeletal Tumors and Diseases

surgery contaminates the femur if a sarcoma is diagnosed and lessens the likelihood of limb

salvage.

RECOMMENDED READINGS

Clayer M, Duncan W. Importance of biopsy of new bone lesions in patients with previous carcinoma. Clin Orthop Relat Res. 2006

Oct;451:208-11. PubMed PMID: 16801861. View Abstract at PubMed

Damron TA, Sim FH. Surgical treatment for metastatic disease of the pelvis and the proximal end of the femur. Instr Course Lect. 2000;49:461-70. Review. PubMed PMID: 10829199.

View Abstract at PubMed

Capanna R, Campanacci DA. The treatment of metastases in the appendicular skeleton. J Bone Joint Surg Br. 2001 May;83(4):471-81.

Review. PubMed PMID: 11380113.

View Abstract at PubMed

Mirels H. Metastatic disease in long bones. A proposed scoring system for diagnosing impending pathologic fractures. Clin Orthop Relat

Res. 1989 Dec;(249):256-64. PubMed PMID: 2684463. View Abstract at PubMed

Figure 117a

Figure 117b

Figure 117c

CLINICAL SITUATION FOR QUESTIONS 117 THROUGH 120

Figures 117a through 117c are the radiographs and MRI scan of a 16-year-old boy who has

had a persistent fullness in his thigh since being kicked while playing soccer 4 weeks ago. He

states that initially the area was painful, but now all symptoms other than the mass have

resolved.

Question 117 of 143

What is the most likely diagnosis?

1- Myositis ossificans

2- Juxtacortical osteogenic sarcoma

Page 85: 2014 Musculoskeletal Tumors and Diseases

3- Osteochondroma

4- Cortical desmoid

PREFERRED RESPONSE: 3- Osteochondroma

Question 118 of 143

You explain to the patient and his parents that the lesion is

1- benign and simply can be observed with serial radiographs.

2- posttraumatic and should be allowed to mature prior to excision.

3- premalignant and should be immediately excised.

4- malignant and should be resected following neoadjuvant chemotherapy.

PREFERRED RESPONSE: 1- benign and simply can be observed with serial radiographs.

Question 119 of 143

A reliable indicator for the presence of malignancy in this scenario would be

1- loss of heterozygosity of the EXT-1 gene.

2- occurrence in a skeletally immature individual.

3- a cartilage cap of 1 cm.

4- growth beyond skeletal maturity.

PREFERRED RESPONSE: 4- growth beyond skeletal maturity.

Question 120 of 143

Findings of multiple lesions in multiple skeletal sites may be associated with

1- decreased risk for malignancy.

2- a characteristic chromosomal translocation.

3- soft-tissue hemangiomas.

4- limb deformity and short stature.

PREFERRED RESPONSE: 4- limb deformity and short stature.

DISCUSSION

This lesion is an osteochondroma. The finding on MRI scan or CT scan of a shared cortex

between the lesion and the host bone is pathognomonic. These lesions are benign and most

often can be followed with serial radiographs. Reasons to excise would be pain, unacceptable

cosmetic deformity, or growth, particularly if the patient is beyond skeletal maturity, which

Page 86: 2014 Musculoskeletal Tumors and Diseases

can be a sign of malignancy. Loss of heterozygosity of either the EXT-1 locus or EXT-2

locus is associated with hereditary multiple exostoses (HME), which increases the risk for

malignant degeneration of 1 or more lesions throughout a patient’s life but does not always

lead to malignancy. Because HME affects all growth plates to some degree, short stature and

limb deformity may occur. A cartilage cap of up to 2 cm can be acceptable, particularly

during youth. If, however, this condition persists or enlarges beyond skeletal maturity,

excision should be entertained.

RECOMMENDED READINGS

Horvai A, Unni KK. Premalignant conditions of bone. J Orthop Sci. 2006 Jul;11(4):412-23. Review. PubMed PMID: 16897210.

View Abstract at PubMed

van der Eijken JW. Strategy in the treatment of benign bone tumors: an overview. J Pediatr Orthop B. 1998 Oct;7(4):249-52. Review.

PubMed PMID: 9810522. View Abstract at PubMed

Vlychou M, Athanasou NA. Radiological and pathological diagnosis of paediatric bone tumours and tumour-like lesions. Pathology. 2008 Feb;40(2):196-216. doi: 10.1080/00313020701813784. Review. PubMed PMID: 18203042.

View Abstract at PubMed

Figure 121a

Figure 121b

Figure 121c

Question 121 of 143

Figure 121a is the axial T1 MRI scan and Figure 121b is the coronal T1 MRI scan of an 85-

year-old man who has a mass in his medial thigh. The mass was present for years and

recently grew. His biopsy specimen is shown in Figure 121c. What is the best treatment for

this patient?

1- Chemotherapy and wide local resection

2- Wide local resection and radiotherapy

3- Marginal excision

4- Observation and reimaging in 6 months

PREFERRED RESPONSE: 2- Wide local resection and radiotherapy

DISCUSSION

The history of a long-standing mass with sudden growth suggests transformation of an

existing tumor. The imaging features of a fatty lesion with internal complexity suggest an

atypical lipomatous tumor (formerly called a well-differentiated liposarcoma). Juxtaposed to

this on imaging is a solid tumor. Histology reveals a high-grade spindle-cell sarcoma adjacent

Page 87: 2014 Musculoskeletal Tumors and Diseases

to a fatty tumor. This clinical presentation is most consistent with a long-standing atypical

lipomatous tumor that has undergone dedifferentiation. The role of chemotherapy for soft-

tissue sarcomas is controversial and unlikely to be applicable in an 85-year-old man.

However, wide local excision with adjuvant radiation therapy is the best option for durable

local control. Marginal excision would be appropriate for an atypical lipomatous tumor that

has not undergone dedifferentiation. Observation of a high-grade sarcoma is not appropriate.

RECOMMENDED READINGS

Nascimento AG. Dedifferentiated liposarcoma. Semin Diagn Pathol. 2001 Nov;18(4):263-6. Review. PubMed PMID: 11757866

View Abstract at PubMed

Murphey MD, Arcara LK, Fanburg-Smith J. From the archives of the AFIP: imaging of musculoskeletal liposarcoma with radiologic-

pathologic correlation. Radiographics. 2005 Sep-Oct;25(5):1371-95. Review. PubMed PMID: 16160117. View Abstract at PubMed

Question 122 of 143

What tumor commonly metastasizes to regional lymph nodes?

1- Fibromyxoid sarcoma

2- Epithelioid sarcoma

3- Leiomyosarcoma

4- Liposarcoma

PREFERRED RESPONSE: 2- Epithelioid sarcoma

DISCUSSION

The common sarcomas that metastasize to regional lymph nodes are epithelioid sarcoma,

synovial sarcoma, angiosarcoma, rhabdomyosarcoma, and clear cell sarcoma. Metastasis to

the lungs remains the most common location for all sarcoma metastases. Daigeler and

associates evaluated regional lymph node metastasis (RLNM) and reported that the condition

was seen in 21.4% of patients with epithelioid sarcoma and 17.6% of patients with clear cell

sarcoma. Other sarcomas had RLNM rates below 10%.

RECOMMENDED READINGS

Daigeler A, Kuhnen C, Moritz R, Stricker I, Goertz O, Tilkorn D, Steinstraesser L, Steinau HU, Lehnhardt M. Lymph node metastases in

soft tissue sarcomas: a single center analysis of 1,597 patients. Langenbecks Arch Surg. 2009 Mar;394(2):321-9. doi: 10.1007/s00423-008-0371-x. Epub 2008 Jul 2. PubMed PMID: 18594854.

View Abstract at PubMed

Maduekwe UN, Hornicek FJ, Springfield DS, Raskin KA, Harmon DC, Choy E, Rosenberg AE, Nielsen GP, DeLaney TF, Chen YL, Ott

MJ, Yoon SS. Role of sentinel lymph node biopsy in the staging of synovial, epithelioid, and clear cell sarcomas. Ann Surg Oncol. 2009

May;16(5):1356-63. doi: 10.1245/s10434-009-0393-9. Epub 2009 Mar 4. PubMed PMID: 19259743. View Abstract at PubMed

Question 123 of 143

A 64-year-old woman has significant right arm pain associated with a destructive proximal

humeral bone lesion with an associated soft-tissue mass. Her medical history includes

Page 88: 2014 Musculoskeletal Tumors and Diseases

carcinoma of the breast treated 8 years ago with modified radical mastectomy, hormone

receptor-based chemotherapy, and 45 Gy of radiation with 8 subsequent disease-free years.

Biopsy reveals a high-grade osteogenic sarcoma. What factor is most likely related to her

current disease?

1- Presence of metastatic disease

2- Dosage of radiation treatment

3- Type of prior surgical procedure

4- Type of chemotherapy given

PREFERRED RESPONSE: 2- Dosage of radiation treatment

DISCUSSION

Postradiation sarcomas are a rare subset of sarcomas (approximately 3% of all sarcomas),

with an incidence lower than 1% in patients receiving high-dose radiation. Diagnosis is

usually based on the following: differing histologic diagnoses between original and

secondary lesion, location within the field of radiation, latent period of more than 4 years, and

radiation doses higher than 40 Gy (although this level is controversial and no true threshold

exists). Chemotherapy is not related to postradiation sarcoma because the ionizing effect of

the radiation causes irreparable cell damage, leading to possible transformation in the future

to malignancies. Five-year survival from postradiation sarcoma is approximately 33%.

Complete surgical resection is the mainstay of treatment, with adjunctive chemotherapy used

in bone sarcomas or metastatic disease. Neither type of surgical procedure nor chemotherapy

has an impact on the development of postradiation sarcoma

RECOMMENDED READINGS

Inoue YZ, Frassica FJ, Sim FH, Unni KK, Petersen IA, McLeod RA. Clinicopathologic features and treatment of postirradiation sarcoma of

bone and soft tissue. J Surg Oncol. 2000 Sep;75(1):42-50. PubMed PMID: 11025461.

View Abstract at PubMed

Kalra S, Grimer RJ, Spooner D, Carter SR, Tillman RM, Abudu A. Radiation-induced sarcomas of bone: factors that affect outcome. J Bone

Joint Surg Br. 2007 Jun;89(6):808-13. PubMed PMID: 17613509. View Abstract at PubMed

Figure 124a

Figure 124b

Figure 124c

Question 124 of 143

Figures 124a and 124b are the MRI scans of a 25-year-old woman who has a painful mass in

her left gluteal and thigh region. Her biopsy specimen is seen in Figure 124c. What is the

most likely diagnosis?

Page 89: 2014 Musculoskeletal Tumors and Diseases

1- Desmoid fibromatosis

2- Extraskeletal Ewing sarcoma

3- Metastatic breast cancer

4- Lymphoma

PREFERRED RESPONSE: 1- Desmoid fibromatosis

DISCUSSION

The imaging shows an infiltrative soft-tissue mass; histology demonstrates a bland spindle-

cell lesion without nuclear atypia consistent with desmoid tumor. Extraskeletal Ewing

sarcoma would be expected to have more surrounding edema and small round blue cells on

histology. Metastatic breast cancer and lymphoma can have similar appearances on MRI scan

but different characteristic histologies, the former with nests of malignant cells, and the latter

with small round blue cells.

RECOMMENDED READINGS

Robbin MR, Murphey MD, Temple HT, Kransdorf MJ, Choi JJ. Imaging of musculoskeletal fibromatosis. Radiographics. 2001 May-

Jun;21(3):585-600. Review. PubMed PMID: 11353108. View Abstract at PubMed

Molloy AP, Hutchinson B, O'Toole GC. Extra-abdominal desmoid tumours: a review of the literature. Sarcoma. 2012;2012:578052. doi: 10.1155/2012/578052. Epub 2012 Aug 16. PubMed PMID: 22966217.

View Abstract at PubMed

Figure 125a

Figure 125b

Figure 125c

CLINICAL SITUATION FOR QUESTIONS 125 THROUGH 128

A 45-year-old woman has increasing knee pain with activity and at rest. Her radiograph, MRI

scan, and histology are shown in Figures 125a through 125c.

Question 125 of 143

The best descriptor for the lesion seen in Figures 125a and 125b is

Page 90: 2014 Musculoskeletal Tumors and Diseases

1- malignant.

2- benign.

3- benign aggressive.

4- inflammatory.

PREFERRED RESPONSE: 3- benign aggressive.

Question 126 of 143

The next appropriate step in staging is

1- biopsy.

2- skeletal survey.

3- whole-body bone scan

4- chest radiograph.

PREFERRED RESPONSE: 4- chest radiograph.

Question 127 of 143

Biopsy results are shown in Figure 125c. Treatment should consist of

1- extended intralesional curettage.

2- chemotherapy and intralesional resection.

3- chemotherapy and complete resection.

4- radiation and complete resection.

PREFERRED RESPONSE: 1- extended intralesional curettage.

Question 128 of 143

The cell that directly causes osteolysis in this lesion is

1- giant cell.

2- stromal cell.

3- osteoblast.

4- fibroblast.

PREFERRED RESPONSE: 1- giant cell.

DISCUSSION

This patient has a giant-cell tumor (GCT) of bone, a benign aggressive lesion. The lesion is

locally aggressive and destroys bone, but metastasizes rarely enough to be considered benign.

Page 91: 2014 Musculoskeletal Tumors and Diseases

The metastatic rate of GCT is 5% or lower, and the metastatic disease most commonly

deposits in the lung, leading to the necessity for systemic staging of the chest. The histology

of a GCT shows large, multinucleated giant cells in a field of smaller stromal cells similar to

the nuclei of the giant cells. The osteoclast-like giant cells are directed through the receptor

activator of nuclear factor kappa-beta ligand pathway to induce lytic bone destruction seen in

this tumor, but the stromal cells are the neoplastic component. The preferred treatment is

complete extended intralesional curettage with high-speed burring and bone grafting or

cementation and fixation as individualized by patient and lesion.

RECOMMENDED READINGS

Turcotte RE. Giant cell tumor of bone. Orthop Clin North Am. 2006 Jan;37(1):35-51. Review. PubMed PMID: 16311110.

View Abstract at PubMed

Kim Y, Nizami S, Goto H, Lee FY. Modern interpretation of giant cell tumor of bone: predominantly osteoclastogenic stromal tumor. Clin

Orthop Surg. 2012 Jun;4(2):107-16. doi: 10.4055/cios.2012.4.2.107. Epub 2012 May 17. Review. PubMed PMID: 22662295. View Abstract at PubMed

Thomas DM. RANKL, denosumab, and giant cell tumor of bone. Curr Opin Oncol. 2012 Jul;24(4):397-403. doi: 10.1097/CCO.0b013e328354c129. Review. PubMed PMID: 22581354.

View Abstract at PubMed

Figure 129a

Figure 129b

Figure 129c

Question 129 of 143

Figures 129a through 129c are the lateral radiograph of the sacrum, axial CT scan, and a

high-power view of a fine-needle biopsy of a 47-year-old man who has buttock pain and

constipation. A large mass is palpable on rectal examination. The tumor cell that is the

signature of this tumor is known as a(n)

1- giant cell.

2- physaliferous cell.

3- chondroblast.

4- adipocyte.

PREFERRED RESPONSE: 2- physaliferous cell.

DISCUSSION

The lateral radiograph and axial CT scan image of the sacrum reveal a destructive lesion in

S4 with a large soft-tissue mass. The differential includes chordoma, giant-cell tumor, and

sacral schwannoma. The images and histology are consistent with chordoma, a tumor usually

diagnosed during the fourth to eighth decades of life. The signature cell of the chordoma is

Page 92: 2014 Musculoskeletal Tumors and Diseases

the physaliphorous cell, although these cells are not pathognomonic of chordoma. Other types

of tumors may have similar-appearing cells, and some chordomas may lack them.

Physaliphorous cells have a large single cytoplasmic vacuole that causes them to appear like

adipocytes. The extracellular stroma is myxoid, resembling chondrosarcoma.

RECOMMENDED READINGS

Nielsen GP, Rosenberg AE. Diagnostic Pathology: Bone. Manitoba, Canada: Amirsys; 2013:8.9-15.

Weber K. Malignant bone tumors. In: Lieberman JR, ed. AAOS Comprehensive Orthopaedic Review. Vol 1. Rosemont, IL: American

Academy of Orthopaedic Surgeons; 2009:417-442.

Figure 130a

Figure 130b

Figure 130c

Figure 130d

Question 130 of 143

Figures 130a through 130c show the CT scan and T1- and T2-weighted MRI scan sequences

of an otherwise healthy 67-year-old woman with progressive left groin pain. Her biopsy

specimen is shown in Figure 130d. Staging studies reveal no other lesions. Treatment should

include

1- radiotherapy.

2- chemotherapy followed by surgical resection.

3- curettage with adjuvant treatment and grafting.

4- en bloc resection.

PREFERRED RESPONSE: 4- en bloc resection.

DISCUSSION

The imaging studies reveal a pelvic chondrosarcoma with extraosseous extension, which is

confirmed by the biopsy specimen. En bloc resection is the treatment of choice for pelvic

chondrosarcomas. Although curettage may be used for select low-grade extremity lesions, it

is not recommended for pelvic lesions. As well, the imaging shows extraosseous extension

and the histology shows a higher-grade lesion (characteristics that make curettage unsuitable

in any skeletal location). Radiotherapy and chemotherapy do not play a role in nonmetastatic

chondrosarcoma.

RECOMMENDED READINGS

Hickey M, Farrokhyar F, Deheshi B, Turcotte R, Ghert M. A systematic review and meta-analysis of intralesional versus wide resection for

intramedullary grade I chondrosarcoma of the extremities. Ann Surg Oncol. 2011 Jun;18(6):1705-9. doi: 10.1245/s10434-010-1532-z. Epub 2011 Jan 22. Review. PubMed PMID: 21258968.

View Abstract at PubMed

Page 93: 2014 Musculoskeletal Tumors and Diseases

Normand AN, Cannon CP, Lewis VO, Lin PP, Yasko AW. Curettage of biopsy-diagnosed grade 1 periacetabular chondrosarcoma. Clin

Orthop Relat Res. 2007 Jun;459:146-9. PubMed PMID: 17452919. View Abstract at PubMed

Figure 131a

Figure 131b

Figure 131c

Question 131 of 143

A lateral radiograph (Figure 131a), sagittal short tau inversion recovery MRI scan (Figure

131b), and an axial T1 contrast MRI scan (Figure 131c) were performed on a 15-year-old boy

who has injured his right knee twice during the last 5 months. He has a reduced range of

motion of the knee and posterior thigh tenderness. A biopsy showed bland spindle cells, giant

cells, and blood-filled spaces without endothelial lining. What is the most appropriate

treatment?

1- Chemotherapy and wide local excision

2- Chemotherapy and radiotherapy

3- Marginal excision

4- Extended intralesional curettage

PREFERRED RESPONSE: 4- Extended intralesional curettage

DISCUSSION

The imaging shows an aneurysmal bone cyst with a destructive, eccentric metaphyseal lesion

with fluid-fluid levels on MR imaging. The described histology confirms this. The

differential diagnosis would include telangiectactic osteosarcoma, although there is no

associated soft-tissue mass on imaging and no malignant stroma described on histology.

Treatment is extended intralesional curettage. Serial embolization is used in surgically

inaccessible spaces. Chemotherapy and/or radiotherapy are not indicated for this benign

lesion. Intralesional treatment rather than excision is the appropriate surgical option.

RECOMMENDED READINGS

Murphey MD, wan Jaovisidha S, Temple HT, Gannon FH, Jelinek JS, Malawer MM. Telangiectatic osteosarcoma: radiologic-pathologic

comparison. Radiology. 2003 Nov;229(2):545-53. Epub 2003 Sep 25. PubMed PMID: 14512511. View Abstract at PubMed

Rapp TB, Ward JP, Alaia MJ. Aneurysmal bone cyst. J Am Acad Orthop Surg. 2012 Apr;20(4):233-41. doi: 10.5435/JAAOS-20-04-233. Review. PubMed PMID: 22474093.

View Abstract at PubMed

Page 94: 2014 Musculoskeletal Tumors and Diseases

Figure 132

CLINICAL SITUATION FOR QUESTIONS 132 THROUGH 134

Figure 132 is the bone scan of a 73-year-old man who is referred from his family doctor with

diffuse bone pain, fatigue, and right knee pain. Examination is notable for pain with motion

about the right knee and mild hyporeflexia.

Question 132 of 143

The most appropriate next step is

1- total spine MRI scan.

2- needle biopsy of the osseous lesion.

3- bone marrow biopsy.

4- CT scan of the chest, abdomen, and pelvis and laboratory studies.

PREFERRED RESPONSE: 4- CT scan of the chest, abdomen, and pelvis and laboratory

studies.

Question 133 of 143

During the course of evaluation, the patient develops a worsening mental status bordering on

stupor. His serum calcium level is 15.8 mg/dL (reference range, 8.5-10.1mg/dL). Treatment

should include

Page 95: 2014 Musculoskeletal Tumors and Diseases

1- initiation of outpatient hemodialysis.

2- cardiac monitoring and fluid restriction.

3- intravenous bisphosphonate treatment.

4- intravenous vitamin D therapy.

PREFERRED RESPONSE: 3- intravenous bisphosphonate treatment.

Question 134 of 143

Further imaging shows pulmonary metastases without an obvious primary tumor of origin

and an incomplete fracture of the right distal femur. A decision is made to surgically treat his

distal femur fracture. What is the role of establishing a preoperative histologic diagnosis for

this patient?

1- The distal femoral lesion should undergo needle biopsy first.

2- Bone marrow biopsy should be done first.

3- Preoperative biopsy is not necessary because a metastatic process is present.

4- Biopsy is not necessary if the lesion demonstrates a standardized uptake value (SUV)

greater than 3 on fluorodeoxyglucose positron emission tomography (PET) imaging.

PREFERRED RESPONSE: 3- Preoperative biopsy is not necessary because a metastatic

process is present.

DISCUSSION

This patient has widespread metastatic disease based on bone scan findings. According to a

study by Rougraff and associates, CT scan of the chest, abdomen, and pelvis and laboratory

studies will identify the primary site of disease in more than 85% of cases. When signs or

symptoms of spinal cord compression are absent, total spine MR imaging probably is not

needed. Biopsy alone is less accurate than staging in establishing a diagnosis. Bone marrow

biopsy is used only if a hematopoietic tumor (myeloma, lymphoma) is diagnosed.

Patients who develop hypercalcemia need hydration (with supplemental furosemide),

bisphosphonates, and evaluation for cardiac arrhythmia. Hemodialysis and vitamin D therapy

are not indicated, and fluid restriction would be contraindicated.

Biopsy before intervention for pathologic lower-extremity fracture likely is not needed in a

scenario with numerous bone lesions and pulmonary metastases. The absence of a clear

primary tumor of origin suggests this is unlikely to be a renal cell carcinoma for which

preoperative embolization would be considered before an intralesional procedure. Tissue

should be obtained at the time of surgery. Although an SUV greater than 3 is suspicious for

malignancy, activity on PET scan should not influence the need for biopsy in this scenario.

RECOMMENDED READINGS

Rougraff BT, Kneisl JS, Simon MA. Skeletal metastases of unknown origin. A prospective study of a diagnostic strategy. J Bone Joint Surg

Am. 1993 Sep;75(9):1276-81. PubMed PMID: 8408149.

View Abstract at PubMed

Page 96: 2014 Musculoskeletal Tumors and Diseases

Biermann JS, Holt GE, Lewis VO, Schwartz HS, Yaszemski MJ. Metastatic bone disease: diagnosis, evaluation, and treatment. J Bone Joint

Surg Am. 2009 Jun;91(6):1518-30. Review. PubMed PMID: 19487533. View Abstract at PubMed

Figure 135a

Figure 135b

Figure 135c

Figure 135d

Figure 135e

Figure 135f

Question 135 of 143

Figures 135a through 135d are the radiographs, sagittal T1-, and coronal T2-weighted MRI

scans of a 22-year-old woman who has had pain in her right ankle for 6 months. Figures 135e

and 135f show a gross photograph of an amputated specimen and a biopsy specimen,

respectively. In this tumor, cytogenetics reveals which translocation?

1- t(12;22)(q13;q12-3)

2- t(X;18)(p11.2;q11.2)

3- t(11;22)(q24;q12)

4- t(1;3)(p36.3;q25)

PREFERRED RESPONSE: 4- t(1;3)(p36.3;q25)

DISCUSSION

The radiographs and MRI scans reveal a multicentric destructive process. Multicentric lesions

in adjacent bones suggest a vascular malignancy such as epithelioid hemangioendothelioma,

which is now included under the category of solitary fibrous tumors. Here, the tumor cells are

in cords within a myxoid stroma that resembles cartilaginous matrix. The translocation that

occurs in epithelioid hemangioendothelioma is t(1;3)(p36.3;q25). Clear cell sarcoma has been

shown to harbor t(12;22)(q13;q12-3), synovial sarcoma t(X;18)(p11.2;q11.2), and Ewing

sarcoma t(11;22)(q24;q12).

RECOMMENDED READINGS

Nielsen GP, Rosenberg AE. Diagnostic Pathology: Bone. Manitoba, Canada: Amirsys; 2013:11.17-21.

Weiss SW, Goldblum JR. Enzinger and Weiss’s Soft Tissue Tumors. 5th ed. Philadelphia, PA: Elsevier; 2008:681-702, 945-988, 1161-1220.

Page 97: 2014 Musculoskeletal Tumors and Diseases

Question 136 of 143

The optimal method to treat a recurrent presentation of pigmented villonodular synovitis

(PVNS) with diffuse joint involvement in a 24-year-old woman with pain and symptomatic

effusions is

1- 700 cGy of radiation therapy.

2- open arthrotomy with synovectomy.

3- imatinib therapy.

4- observation.

PREFERRED RESPONSE: 2- open arthrotomy with synovectomy.

DISCUSSION

Local control of recurrent diffuse PVNS is best accomplished with open arthrotomy and

complete synovectomy. Arthroscopic methods in the setting of persistent diffuse PVNS are

associated with an unacceptably high rate of recurrence. Radiation therapy (700 cGy) is the

typical dose administered for heterotopic ossification prophylaxis, but this dose is not high

enough to achieve local control in PVNS. Imatinib, a colony-stimulating factor inhibitor, has

been described for recurrent/refractory disease, but is not considered as effective as open

surgical treatment.

RECOMMENDED READINGS

Tyler WK, Vidal AF, Williams RJ, Healey JH. Pigmented villonodular synovitis. J Am Acad Orthop Surg. 2006 Jun;14(6):376-85. Review.

PubMed PMID: 16757677.

View Abstract at PubMed

Heyd R, Micke O, Berger B, Eich HT, Ackermann H, Seegenschmiedt MH; German Cooperative Group on Radiotherapy for Benign

Diseases. Radiation therapy for treatment of pigmented villonodular synovitis: results of a national patterns of care study. Int J Radiat Oncol Biol Phys. 2010 Sep 1;78(1):199-204. doi: 10.1016/j.ijrobp.2009.07.1747. Epub 2010 May 18. PubMed PMID: 20488628.

View Abstract at PubMed

Cassier PA, Gelderblom H, Stacchiotti S, Thomas D, Maki RG, Kroep JR, van der Graaf WT, Italiano A, Seddon B, Dômont J, Bompas E,

Wagner AJ, Blay JY. Efficacy of imatinib mesylate for the treatment of locally advanced and/or metastatic tenosynovial giant cell

tumor/pigmented villonodular synovitis. Cancer. 2012 Mar 15;118(6):1649-55. doi: 10.1002/cncr.26409. Epub 2011 Aug 5. PubMed PMID: 21823110.

View Abstract at PubMed

Page 98: 2014 Musculoskeletal Tumors and Diseases

Figure 137a

Figure 137b

Question 137 of 143

The radiographs of a 63-year-old man with knee and proximal tibia discomfort are shown in

Figures 137a and 137b. What genetic abnormality is associated with this condition?

1- Fibrillin mutation

2- EXT 1 gene

3- p53 mutation

4- t(x;18)

PREFERRED RESPONSE: 2- EXT 1 gene

DISCUSSION

The radiographs reveal multiple osteochondromas consistent with multiple hereditary

exostoses. The EXT 1 and EXT 2 genes have been associated with this condition. The EXT 1

gene is located at 8q24 and the EXT 2 gene at 11 p11-p12, but additional linkage to

chromosomal locus 19p has suggested the existence of an as of yet unidentified EXT 3 gene.

Fibrillin mutations are associated with Marfan syndrome, p53 mutations with a variety of

malignancies, and t(x;18) with synovial sarcoma.

RECOMMENDED READINGS

Congenital and inherited bone conditions. In: Damron TA, ed. Orthopedic Surgery Essentials: Oncology and Basic Science. Philadelphia, PA: Lippincott Williams and Wilkins; 2008:212-228.

Bovée JV. Multiple osteochondromas. Orphanet J Rare Dis. 2008 Feb 13;3:3. doi: 10.1186/1750-1172-3-3. Review. PubMed PMID:

18271966.

View Abstract at PubMed

Bridge JA, Nelson M, Orndal C, Bhatia P, Neff JR. Clonal karyotypic abnormalities of the hereditary multiple exostoses chromosomal loci

8q24.1 (EXT1) and 11p11-12 (EXT2) in patients with sporadic and hereditary osteochondromas. Cancer. 1998 May 1;82(9):1657-63.

PubMed PMID: 9576285.

View Abstract at PubMed

RESPONSES FOR QUESTIONS 138 THROUGH 143

Page 99: 2014 Musculoskeletal Tumors and Diseases

1- Multiple myeloma

2- Prostate cancer

3- Adenocarcinoma of the breast

4- Adenocarcinoma of the lung

5- Thyroid cancer

6- Renal cell carcinoma

Match the descriptions below to the conditions listed above.

Question 138 of 143

Often associated with the shortest mean survival once it is metastatic to bone and solid organs

1- Multiple myeloma

2- Prostate cancer

3- Adenocarcinoma of the breast

4- Adenocarcinoma of the lung

5- Thyroid cancer

6- Renal cell carcinoma

PREFERRED RESPONSE: 4- Adenocarcinoma of the lung

Question 139 of 143

Advanced disease including bone metastases is treated with vascular endothelial growth

factor (VEGF) inhibitors (ie, sunitinib) and, typically, the lesions are radioresistant.

1- Multiple myeloma

2- Prostate cancer

3- Adenocarcinoma of the breast

4- Adenocarcinoma of the lung

5- Thyroid cancer

6- Renal cell carcinoma

PREFERRED RESPONSE: 6- Renal cell carcinoma

Question 140 of 143

Often treated with bisphosphonates and radioiodine in the setting of metastatic bone disease

1- Multiple myeloma

2- Prostate cancer

3- Adenocarcinoma of the breast

4- Adenocarcinoma of the lung

5- Thyroid cancer

6- Renal cell carcinoma

Page 100: 2014 Musculoskeletal Tumors and Diseases

PREFERRED RESPONSE: 5- Thyroid cancer

Question 141 of 143

Frequently involves mixed osteolytic and osteoblastic metastatic bone disease that typically is

less responsive to treatment if the patient is human epidermal growth factor receptor 2 (HER-

2)-positive

1- Multiple myeloma

2- Prostate cancer

3- Adenocarcinoma of the breast

4- Adenocarcinoma of the lung

5- Thyroid cancer

6- Renal cell carcinoma

PREFERRED RESPONSE: 3- Adenocarcinoma of the breast

Question 142 of 143

Classically known for predominantly osteoblastic metastatic bone disease in male patients

1- Multiple myeloma

2- Prostate cancer

3- Adenocarcinoma of the breast

4- Adenocarcinoma of the lung

5- Thyroid cancer

6- Renal cell carcinoma

PREFERRED RESPONSE: 2- Prostate cancer

RESPONSES FOR QUESTIONS 138 THROUGH 143

1- Multiple myeloma

2- Prostate cancer

3- Adenocarcinoma of the breast

4- Adenocarcinoma of the lung

5- Thyroid cancer

6- Renal cell carcinoma

Match the descriptions below to the conditions listed above.

Question 143 of 143

A small round blue-cell neoplasm of plasma cells responsive to chemotherapy and radiation

Page 101: 2014 Musculoskeletal Tumors and Diseases

1- Multiple myeloma

2- Prostate cancer

3- Adenocarcinoma of the breast

4- Adenocarcinoma of the lung

5- Thyroid cancer

6- Renal cell carcinoma

PREFERRED RESPONSE: 1- Multiple myeloma

DISCUSSION

Metastatic bone disease is a common phenomenon. The 5 most common primary cancers that

spread to bone are breast, lung, thyroid, kidney, and prostate (commonly remembered with

the easy pneumonic BLT with a Kosher Pickle). Radiologically, lung, thyroid, and kidney

cancers are mostly radiolucent, prostate cancer classically shows osteoblastic changes, and

breast cancer demonstrates a mix. Lung cancer in general is associated with the shortest mean

survival among the primary carcinomas listed once the disease is metastatic to multiple

locations including bone. Patients generally survive less than 1 year after diagnosis.

Metastatic renal cell carcinoma has a poor prognosis, but a small subset of patients can

survive for years. Newer drugs such as VEGF inhibitors have shown in select cases to be

effective, presumably by diminishing blood supply to these vascular tumors. These tumors

are often radioresistant, making them difficult to control with nonsurgical measures. Thyroid

cancer that has spread to bone is treated like most metastatic bone cancer. Bisphosphonates

are used to help reduce bone pain and decrease risk for pathologic fracture. However,

radioiodine is often used as targeted therapy. Prostate cancer that metastasizes to bone is best

known for its classic osteoblastic lesions. However, it is important to note that this is not

always the case. Whole-body bone scans often show intense uptake at the site of the

metastatic lesions in patients with prostate cancer, as opposed to those with renal cell

carcinoma or multiple myeloma, who often demonstrate little bony reaction and have a

“cold” scan. Multiple myeloma is a primary cancer of the bone marrow. It is classified as a

small round blue-cell tumor histopathologically and is generally responsive to chemotherapy

and radiation. Metastatic breast cancer often features lesions that are mixed radiolucent and

osteoblastic. The response to treatment is often dictated by tumor cell surface markers.

Typically, the most responsive tumors are estrogen and progesterone receptor positive and

HER-2 negative. HER-2-positive tumors denote a more aggressive and difficult to treat

subset of cancer.

RECOMMENDED READINGS

Biermann JS, Holt GE, Lewis VO, Schwartz HS, Yaszemski MJ. Metastatic bone disease: diagnosis, evaluation, and treatment. J Bone Joint Surg Am. 2009 Jun;91(6):1518-30. Review. PubMed PMID: 19487533.

View Abstract at PubMed

Lewis VO. What's new in musculoskeletal oncology. J Bone Joint Surg Am. 2009 Jun;91(6):1546-56. doi: 10.2106/JBJS.I.00375. Review.

PubMed PMID: 19487537.

View Abstract at PubMed

Schmidinger M, Larkin J, Ravaud A. Experience with sunitinib in the treatment of metastatic renal cell carcinoma. Ther Adv Urol. 2012

Oct;4(5):253-65. PubMed PMID: 23024706. View Abstract at PubMed

Rossi A, Gridelli C, Ricciardi S, de Marinis F. Bone metastases and non-small cell lung cancer: from bisphosfonates to targeted therapy. Curr Med Chem. 2012 Nov 1;19(32):5524-35. PubMed PMID: 22963665.

View Abstract at PubMed

Page 102: 2014 Musculoskeletal Tumors and Diseases

Lopez-Olivo MA, Shah NA, Pratt G, Risser JM, Symanski E, Suarez-Almazor ME. Bisphosphonates in the treatment of patients with lung

cancer and metastatic bone disease: a systematic review and meta-analysis. Support Care Cancer. 2012 Nov;20(11):2985-98. doi: 10.1007/s00520-012-1563-z. Epub 2012 Sep 7. PubMed PMID: 22956190.

View Abstract at PubMed

This is the last question of the exam.